matematik olimpiyatlarına hazırlık 1

advertisement
MATEMATİK
OLİMPİYATLARINA
HAZIRLIK
1
TEMEL BİLGİLER -I
Mustafa Özdemir
ALTIN NOKTA YAYINEVİ
İZMİR - 2016
Copyright © Altın Nokta Basım Yayın Dağıtım Bilişim
ISBN 978-975-6146-62-0
MATEMATİK OLİMPİYATLARINA HAZIRLIK-1 TEMEL BİLGİLER-I
Mustafa Özdemir
[email protected]
Bu kitabın her hakkı saklıdır. Tüm hakları Altın Nokta Basım Yayın Dağıtım'a aittir. Kısmen de
olsa alıntı yapılamaz. Metin ve sorular, kitabı yayımlayan kurumun önceden izni olmaksızın
elektronik, mekanik, fotokopi ya da herhangi bir kayıt sistemiyle çoğaltılamaz, yayımlanamaz.
Bu kitaptaki TÜBİTAK Matematik Olimpiyat Soruları, TÜBİTAK – Bilim İnsanı Destekleme Daire
Başkanlığından izin alınarak yayımlanmaktadır.
Genel Yayın Yönetmeni
Halil İ. AKÇETİN
Kapak-Dizgi
Altın Nokta Dizgi-Grafik
Baskı
ERTEM BASIM YAYIN DAĞ. SAN. TİC.LTD. ŞTİ.
Nasuh Akar Mah. 25. Sok. No: 19 Çankaya / ANKARA
Tel: 0 (312) 640 16 23
Yayın - Dağıtım
Altın Nokta Basım Yayın Dağıtım
859 Sk. No:1/Z-4 Konak / İZMİR
Tel- Faks : 0 (232) 441 25 95
www.nokta2000.com
www.altinnokta.com.tr
www.kitapana.com
[email protected]
[email protected]
[email protected]
Ocak - 2016
5. Basım
Önsöz
Antalya Matematik Olimpiyatlarına öğrenci hazırlayan bazı öğretmenlerimizin
Türkçe kaynak bulamamaktan şikayet etmeleri üzerine yazmaya karar verdiğim ve
bir kitaptan oluşacağını düşünerek başladığım olimpiyatlara hazırlık kitabı, bir anda
beş ciltlik güzel bir kaynak oldu. Bu kitaplarda, hemen hemen her konuya ve bu
konularla ilgili değişik sorulara yer vermeye çalıştım.
Aslında, İngilizce bilen her öğrenci internette birçok olimpiyat kaynağına ulaşa­
bilir. Fakat, konularına göre yazılmış ve konularına göre sorular verilmiş Matematik
Olimpiyatlarına Hazırlık Kitapları, olimpiyatlara yeni başlayan öğrenciler ve olim­
piyatlara öğrenci hazırlayan öğretmenler için büyük bir kolaylık sağlamıştır. Bu ki­
taplardan öğrenilen temel bilgiler sayesinde, farklı dillerden ve farklı kaynaklardan
ulaşılan bir çok olimpiyat sorusunun ve çözümünün daha çabuk anlaşılır hale gele­
ceğini umuyorum.
Kitaplarla ilgili Türkiye’nin hemen hemen her yerinden hem öğrencilerden hem
de öğretmenlerimizden teşekkür e­postaları ve telefonları almaktayım. Türkiye’de
böyle bir kaynağın olmasından dolayı teşekkür eden birçok öğrenci ve öğretmeni­
miz, konularına göre soruların verilmiş olmasının, her konunun detaylı şekilde in­
celenmiş olmasının ve anlaşılır bir dille yazılmış olmasının, kendilerine büyük ko­
laylık sağladığını e­postalarında ifade ediyorlar. Kitapta bulunan bazı hataları da bana
bildiren öğrenci ve öğretmenlerimiz sayesinde, daha sonraki basımlarda, bu hataların
giderilmesine çaba sarf ediyorum. Her ne kadar tashihini yapmış olsam da, soruların
zorluğu ve çokluğu ve zamanın darlığı bazen hataları görmemize engel oluyor. Ki­
taplardaki olabilecek diğer hatalarımı ve kitapla ilgili görüş ve düşüncelerinizi, yine
[email protected] e­posta adresine gönderirseniz sevinirim.
Bu kitaplarla güzel ülkeme çok küçük de olsa, bir faydam olmuşsa, bu benim
için büyük bir mutluluktur.
Hangi Ciltte Hangi Konular Var?
Birinci ve ikinci ciltte, olimpiyatlar için en gerekli temel kavramların ve yön­
temlerin verilmesi amaçlandı. Bunun için, temel kavramlar, tanımlar, gösterimler
verilerek, problem tipleri, çarpanlara ayırma, çözümleme, toplamlar, kombinatorik,
binom açılımı, ispat yöntemleri konuları ele alındı.
Üçüncü ciltte ise, sayılar teorisi konusu ele alınarak, bölünebilme, asal sayılar,
obeb­okek, modüler aritmetik, Fermat, Euler, Wilson teoremleri, Çin kalan teoremi,
denklikler, tamdeğer, konuları verildi.
Dördüncü ciltte ise, fonksiyonlar, polinomlar, polinom denklemler ve eşitsizlik­
ler, diziler, denklemler ve denklem sistemleri konularına yer verildi.
Beşinci cillte ise, logaritma ve trigonometri bilgisi, limit, süreklilik, türev, fonksi­
yonel denklemler ve eşitsizlikler konuları verildi.
Her bir kitapta öncelikle, konuya ve o konu ile ilgili örnek öğretici olabilecek
sorulara yer verdim. Daha sonra, her bir konu ile ilgili dünyada değişik olimpiyatlarda sorulmuş soruları ve çözümlerini ilave ederek, öğrencilerin karşılaşabileceği
farklı soruları görmesini sağladım.
Türkiye’deki Matematik Olimpiyatları Konusunda Kısa Bilgi
Türkiye’de olimpiyat etkinlikleri, TÜBİTAK Bilim İnsanı Destekleme Daire
Başkanlığı (BİDEB) tarafından yürütülmektedir. Bu çalışmalar hem ulusal düzeyde
hem de uluslararası düzeyde yapılmaktadır. Ulusal düzeyde gerçekleştirilen İlköğre­
tim Matematik Olimpiyatı ile Liseler İçin Matematik Olimpiyatları sonuçlarına göre
ülkemizi Uluslararası yarışmalarda temsil edecek takımlar belirlenmektedir. Ulus­
lararası Bilim Olimpiyatlarında ülkemizi temsil edecek takımlar matematik olimpiyat
kamplarında başarılı olmuş öğrencilerin, çeşitli sınavlar sonucunda seçilmeleriyle oluş­
maktadır. Şu ana kadar katıldığımız Uluslararası Matematik Olimpiyatlarında,
Umut Varolgüneş, Melih Üçer, Ömer Faruk Tekin, Cafer Tayyar Yıldırım, Selim Ba­
hadır (2 kez), Nizameddin Ordulu, Mehmet Bumin Yenmez ülkemize altın madalya
kazandıran öğrencilerdir.
Son yıllarda, birçok üniversite lise öğrencilerine yönelik olarak matematik olim­
piyatları düzenlemektedir. Bunlardan en eskisi Akdeniz Üniversitesi tarafından düzen­
lenen Ulusal Antalya Matematik Olimpiyatlarıdır, bu olimpiyat birincisi test ve ikin­
cisi klasik olarak iki aşamada yapılmaktadır. Yine, Fatih, Koç, Doğuş, Mersin, Sa­
bancı üniversiteleri de matematik olimpiyatı düzenleyen üniversitelerden bazılarıdır.
Matematik Olimpiyatlarına Hazırlanan Bir Öğrenci Ne Kazanır?
Matematik olimpiyatlarına hazırlanmak hem zor hem de zevklidir. Matematik
olimpiyatlarına hazırlanan bir öğrenci sınavın sonucunda hangi dereceyi alırsa alsın
asla kaybetmez. Öğrendiği konular ve zor soruların yanında, beynini zorlaması
ufkuna açmasına ve ileride zor problemler ile karşılaştığında daha sağlıklı ve daha
tutarlı yorumlar yapmasını sağlayacaktır. Sporla uğraşan bir sporcu katıldığı olim­
piyatta başarılı olamasa bile, hazırlanma aşamasında vücudunun sağlıklı olması için
yaptığı çalışmaların faydasını gördüğü gibi, matematik olimpiyatlarına hazırlanan bir
öğrenci de, zor problemlere kafa yormasının sonucu olarak beynini geliştirir. İnsan­
lar düşündükçe aklını kullandıkça , matematik problemi çözdükçe beyin hücrelerinin
yolları açılır. Bilim adamları, normal insanların mevcut beyin kapasitelerinin çok
az bir kısmını kullanabildiğini söylemektedirler. Bu kapasite elbette sıradan işlerle
uğraşarak, beyni yormayarak, basit ve birbirine benzeyen problemleri çözerek artma­
yacaktır. Beyni yormak gerekir. Beyni zorlamak, sürekli yeni problemlerle meşgul
etmek gerekir. Beyin hücreleri kullanılmaz ise kaybedilir. O halde, bir matematik
yarışmasına girsek de girmesek de zor sorular ile uğraşmalıyız.
Matematik Olimpiyatlarına Nasıl Hazırlanılmalı?
Matematik Olimpiyatlarına hazırlanmak gerçekten zordur. Zaman ister. Tıpkı
olimpiyata hazırlanan bir haltercinin sürekli kendini geliştirmesi, yavaş yavaş ağır­
lıkları kaldırması ve bunu başarabilmek içinde gerekli zamanı harcayıp vücudunu
geliştirmesi gibi, yavaş yavaş ilerlenmesi gereken bir çalışmadır. Olimpiyat sorularını
çözmeye yeni başlayan birisine, bazı soruların oldukça zor gelmesi normaldir. Bu bi­
raz bilgiye, biraz tecrübeye biraz da püf noktalı sorulara hazırlıklı olmaya göre değişir.
Soruların zorluk derecesi, elbetteki, bir halterin ağırlığı gibi net olarak ifade edilmese
de, bildiğiniz bir konuda sorulan bir sorudaki ince bir püf nokta o soruyu çok zor hale
getirebilir. Bir soru öğrenildikten sonra kolaydır. Öğreninceye kadar zor bir sorudur.
Bu kitabın amaçlarından biri de size göre zor olan soruların sayısının azalmasına
yardımcı olmaktır. Olimpiyatlara hazırlanan bir öğrenci herşeyden önce, kararlı
olmalı, kendine güvenmeli, fakat ne kadar kendine güvenirse güvensin yapamaya­
cağı soruların olduğunun farkında olup, çözemediği sorular karşısında umutsuzluğa
düşmek yerine, çözemediği soruların çözümlerini öğrenerek ilerlemesi gerektiğinin
bilincinde olmalıdır. Kısaca, matematik olimpiyatlarına hazırlık, kararlılık, sabır ve
azim isteyen bir iştir. Acele etmemek gerekir. Hatta bazı soruların çözümü de an­
laşılamayabilir veya bir sorunun çözümü öğrenildikten sonra tekrar karşılaşıldığında
o soruyu yapamayabilirsiniz. Öğrencilerden, bu konu ile ilgili en çok karşılaştığım
soru, "çözümünü gördüğümüz zaman anlıyoruz ama kendimiz yapamıyoruz, ne yap­
malıyız?" sorusudur. Aslında bu normaldir. Olimpiyat sorularının kendine has çözme
yöntemleri olabilir. Bu yöntemleri bir anda öğrenmek elbette kolay değildir. Bu
kitapta konular ve konu ile ilgili sorulan sorular mümkün olduğu kadar, o konuya
gelinceye dek öğrenilen bilgileri içerecek şekilde ele alınmıştır. Bir soruyu çözerken,
soruyu önce kendiniz çözmeye çalışınız. Çözemez iseniz, çözümünü inceleyip
nasıl bir yöntem kullanıldığını inceleyiniz ve soruda püf nokta var ise, o püf noktayı mutlaka görmeden soruyu geçmeyiniz. Sorunun çözümünü anlamaz iseniz, bu
konu ile ilgili bilgilerinizin eksik olabileceğini göz önünde bulundurarak umutsuzluğa
kapılmayınız. Unutmayın sizi zorlayan her soru sizin için zor ve güzel bir sorudur.
Bazı sorularda hata da olabilir. Bu tür hataları bildirirseniz, kitabın bundan sonraki
basımlarında daha hatasız olarak size ulaştırabiliriz.
Kısa Özgeçmiş
Mustafa Özdemir, 1975 yılında Konya’nın Bozkır ilçesinde doğdu. İlk, orta ve lise
öğrenimini Antalya’da tamamladı. 1992 yılında girdiği Dokuz Eylül Üniversitesi
Buca Eğitim Fakültesi Matematik Öğretmenliği Bölümü’nden 1996 yılında mezun
oldu. 1999 ­ 2007 yılları arasında, Akdeniz Üniversitesi, Fen Bilimleri Enstitüsü,
Matematik Anabilim Dalında yüksek lisans ve doktorasını tamamladı. Halen, Akde­
niz Üniversitesi Matematik Bölümünde çalışmaktadır.
Beşinci Baskı İçin Teşekkür.
Kitabın daha önceki baskılarında hatalı soru çözümleri, baskı hataları, eksik
çözümler, yanlış ifade edilişler ile ilgili birçok hatalarımı görüp bildiren, başta Bahçe­
şehir Üniversitesi Öğretim Üyesi Yrd. Doç. Dr. Maksat Ashyraliyev olmak üzere,
Başer Kandehir’e, Reşit Kaya’ya, Mahmut Bektaş’a, Taha Eyüp Korkmaz’a,
Salih Can’a, Ahmet Arduç’a, Ebubekir Celayir’e, Oğuzhan Yılmaz’a, Ahmet
Alaydın’a ve Mustafa Küçük’e çok teşekkür ediyorum. Onların da katkılarıyla kitap
çok daha hatasız hale gelmiştir.
Ayrıca, Matematik Olimpiyatlarına Hazırlık kitaplarını yazmamda bana destek
olan değerli hocalarım, Prof. Dr. Abdullah Aziz Ergin’e, Prof. Dr. Halil İbrahim
Karakaş’a, Prof. Dr. İlham Aliyev’e, Prof. Dr. Hasan Ali Çelik’e ve Prof. Dr. Ali
Nesin’e ve eşim Burcu Özdemir’e teşekkür ederim.
Son olarak, Altın Nokta Yayınevi olarak, kitabı yayınları arasına alarak basımı ve
dağıtımı konusunda her türlü fedakarlığı yapan Halil İbrahim Akçetin’e ve değerli
eşi Leyla Akçetin’e çok teşekkür ederim.
Mustafa Özdemir
Antalya ­ 2015
[email protected]
Hayatta iken değeri yeterince bilinmeyen tüm anneler adına
Annem Hayriye Özdemir’e
İçindekiler
BİRİNCİ BÖLÜM
Temel Bilgiler
Sayılar
Ardışık Sayıların Toplamı
Bölünebilme Kuralları
Bir Sayının Pozitif Bölenlerinin Sayısı, Toplamı ve Çarpımı
EBOB ­ EKOK
Modüler Aritmetik
Basit Denklem Çözümleri ve Sayıların Özelliklerinin Kullanılması
Basit Eşitsizlikler
Faktöriyel Kavramı
Bir Sayının Tam Kısmı
Mutlak Değer
Üslü ve Köklü Sayılar
Oran ­ Orantı
Karışık Örnekler
Çözümlü Test 1
Çözümler
TÜBİTAK SORULARI (Temel Bilgiler)
TÜBİTAK SORULARININ ÇÖZÜMLERİ (Temel Bilgiler)
ULUSAL ANTALYA MATEMATİK OLİMPİYATI SORULARI
11
16
19
25
29
34
38
44
46
50
54
58
64
65
73
78
87
99
119
İKİNCİ BÖLÜM
Problemler
Yaş Problemleri
İşçi ­ Havuz Problemleri
Hareket Problemleri
Yüzde ­ Faiz Problemleri
Karışım Problemleri
Saat Problemleri
Sınav Problemleri
Tahtadaki Sayı Problemleri
125
127
131
133
136
137
138
139
Tartı Problemleri
Sayı Tablosu ve Sihirli Kare Problemleri
Mantık Problemleri
143
145
147
Oyun ve Turnuva Problemleri
Çember Etrafına Sayı Yerleştirme Problemleri
Çözümlü Test 2
Çözümler
TÜBİTAK SORULARI (Problemler)
TÜBİTAK SORULARININ ÇÖZÜMLERİ (Problemler)
ULUSAL ANTALYA MATEMATİK OLİMPİYATI SORULARI
148
151
155
160
167
181
203
ÜÇÜNCÜ BÖLÜM
Çarpanlara Ayırma ve Özdeşlikler
Çarpanlara Ayırma Yöntemleri
Özdeşlikler
Karışık Örnekler
Çözümlü Test 3
Çözümler
TÜBİTAK SORULARI (ÇarpanlaraAyırma)
TÜBİTAK SORULARININ ÇÖZÜMLERİ (Çarpanlara Ayırma)
ULUSAL ANTALYA MATEMATİK OLİMPİYATI SORULARI
207
214
230
239
244
255
261
273
DÖRDÜNCÜ BÖLÜM
Çözümleme ve Taban Aritmetiği
Çözümleme
Rakam Değiştirme veya Silme
Karışık Örnekler
Çözümlü Test 4
Çözümler
TÜBİTAK SORULARI (Çözümleme)
TÜBİTAK SORULARININ ÇÖZÜMLERİ (Çözümleme)
ULUSAL ANTALYA MATEMATİK OLİMPİYATI SORULARI
275
281
284
295
302
317
323
335
BEŞİNCİ BÖLÜM
Eşitsizliklere Giriş
Aritmetik ­ Geometrik ­ Harmonik Ortalama ve Eşitsizlikleri
337
Cauchy ­ Schwartz Eşitsizliği
Karışık Örnekler
Çözümlü Test 5
Çözümler
ULUSAL ANTALYA MATEMATİK OLİMPİYATI SORULARI
YANIT ANAHTARI
346
350
352
355
360
364
Temel Bilgiler
1.1
Sayılar
I N = {0 1 2 3 } kümesinin her bir elemanına doğal sayı denir.
Z = { −2 −1 0 1 2 } kümesinin her bir elemanına tamsayı denir.
Z+ = {1 2 3 } kümesine pozitif tamsayılar kümesi ve
Z− = { −3 −2 −1} kümesine de negatif tamsayılar kümesi denir.
Sıfır bir tamsayıdır, fakat pozitif veya negatif değildir.

I  ve  tamsayı ve  6= 0 olmak üzere, şeklinde yazılabilen sayılara rasyonel

sayı denir. Rasyonel sayılar kümesini,
o
n
:   ∈ Z ve  6= 0
Q=

ile gösteririz. Virgülden sonraki bir kısımı periyodik olarak devam eden rasyonel
sayılara devirli ondalık sayılar denilir. Devreden kısım üstü çizgili olarak gösterilir.
Devirli bir ondalık sayıyı rasyonel olarak ifade edebiliriz. Bunun için, verilen devirli
sayı, devreden kısım sadece bir kez yazılmak şartıyla, virgülsüz kabul edilir. Sonra, bu
sayıdan, sürekli tekrar eden devirli kısım haricindeki sayı çıkarılır. Elde edilen sayı,
virgülden sonraki devreden kadar 9 devretmeyen kadar 0’ın yanyana yazılmasıyla elde
 − 
edilen sayıya bölünür. Örneğin,   devirli sayısı,
olarak yazılır.
9990

I Rasyonel olmayan sayılara irrasyonel sayılar denir. Yani,
şeklinde yazı­

lamayan, virgülden sonrası
√ √düzenli bir biçimde tekrarlanmayan sayılara irrasyonel
sayılar denir. Örneğin, 2, 3 ve  sayılarının kesirli olmadıkları bilinir. İlk ikisinin
kanıtı kolaysa da üçüncüsünün kanıtı zordur. İrrasyonel sayıları, Q0 ile göstereceğiz.
I Rasyonel sayılar kümesiyle irrasyonel sayıların kümesinin birleşimine reel (gerçel)
sayılar kümesi denir. R = Q ∪ Q0 ile gösterilir.
I Bir rasyonel sayıyla bir irrasyonel sayının toplamı irrasyoneldir. Bunu göstere­
lim.
 ∈ Q ve  ∈ Q0 olsun.  +  =  ∈ Q olsaydı,  =  −  eşitliğine göre, iki
rasyonel sayının farkı yine bir rasyonel sayı olacağından,  sayısı da rasyonel olurdu.
Fakat,  sayısını bir irrasyonel sayı kabul etmiştik. O halde, bir rasyonel sayıyla bir
irrasyonel sayının toplamı rasyonel olamaz.
√
√
I İki irrasyonel sayının toplamı irrasyonel sayı olmayabilir. Örneğin, 2 ve 1 − 2
sayıları irrasyonel sayılar olmasına rağmen toplamları 1’dir ve rasyoneldir.
√
√
I İki irrasyonel sayının çarpımı irrasyonel sayı olmayabilir. Örneğin, 3 ve 3
irrasyonel sayılarının çarpımı 3’dür ve rasyoneldir.
I Bir rasyonel sayı ile bir irrasyonel sayının çarpımı irrasyoneldir denilemez. Ras­
yonel sayılardan birini, 0 alırsak, sonuç 0, yani bir rasyonel sayı olur.
12
Matematik Olimpiyatlarına Hazırlık 1
I [ ] gösterimi  ve  dahil  ile  arasındaki reel sayıları gösterir. Yani,  ∈ [ ]
ise,  ≤  ≤ ’dir. ( ] gösterimi,  hariç ve  dahil  ile  arasındaki reel sayıları
gösterir. Yani,  ∈ ( ] ise,    ≤  olur ve eğer      ise,  ∈ ( )’dir.
I  bir tamsayı olmak üzere, 2 şeklinde ifade edilen sayılara çift sayı, 2 − 1
şeklinde ifade edilen sayılara da tek sayı denir. Çift sayıları, Ç ile, tek sayıları da 
ile göstereceğiz.
 ±  = Ç,  ± Ç =  Ç ± Ç = Ç,
 ·  =   · Ç = Ç, Ç · Ç = Ç,
+
ve  ∈ Z için
  =  , Ç = Ç
biçimindedir.
I 1 ve kendisinden başka pozitif tam böleni olmayan, 1’den büyük tamsayılara asal
sayılar denir. 1 asal sayı değildir. 2’den başka çift asal sayı yoktur.
√
Bir  sayısının asal olup olmadığını nasıl anlamak için ’yi ’den küçük asal
sayılara bölerek√kontrol edebiliriz. Çünkü  =  ve  ≤  ise  ≥ ’dir. Dolayısıyla
 asal değilse ’den küçük bir asal sayıya bölünür.
√
Örneğin, 199 sayısı asaldır. Çünkü, 199 ' 142 olduğundan 14’ten küçük 2
3 5 7 11 ve 13 sayılarının hiçbiri 199 sayısını bölmez.
Örnek 1 3n − 10, 6n − 13 ve 5n − 13 sayılarının üçü de asal sayı olacak şe­
kilde kaç  pozitif tamsayısı vardır?
Çözüm : Verilen üç sayının toplamı,
(3 − 10) + (5 − 13) + (6 − 13) = 14 − 36
olduğundan çifttir. Üç asal sayının toplamının çift olabilmesi için birinin mutlaka 2
olması gerekir. Buna göre,
3 − 10 = 2 ise,  = 4 olacağından, 5 − 13 = 7 ve 6 − 13 = 11 olduğundan
üçü de asal olur.
5 − 13 = 2 ise,  = 3 olur, fakat, 3 − 10 asal olmaz.
6 − 13 = 2 olması ise mümkün değildir.
O halde, sadece  = 4 değeri için üç sayıda asal sayıdır.
Örnek 2 p, p + 10 ve p + 14 sayıları asal olacak şekilde kaç p sayısı vardır?
Çözüm : Bu sayıların 3’e bölümünden kalanlar sırasıyla, ’nin,  + 1’in ve  + 2’nin
3’e bölümünden kalanlara eşittir ve bu kalanlar birbirinden farklıdır. Bu durumda, bu
sayılardan biri 3’e bölünmelidir. Sayılar asal olduğundan, en küçük olan  sayısının 3
olması gerekir. Böylece diğer sayılar 13 ve 17 olur.
13
Temel Bilgiler
Örnek 3 100 tane pozitif tamsayının toplamı çarpımından büyüktür. Bu sayılar­
dan en fazla kaçı 1’den büyük olabilir?
Çözüm : 1’den farklı pozitif sayıların en çok olması için, 1’den büyük en küçük
tamsayı olan 2 sayısını kullanmalıyız. Daha büyük sayıları kullanmamız çarpımın
daha hızlı büyümesine neden olur. Buna göre, 7 tane 2 kullanmış olsak, geriye 93
tane 1 kalır ki, bunların çarpımı 27 = 128 iken toplamları, 93 + 7 · 2 = 107 olur ve
koşul sağlanmaz. Bu nedenle, 2 sayısı 7’den küçük olmalıdır. 6 tane 2 olursa, çarpım
26 = 64 ve toplam ise 94 + 6 · 2 = 106 olur. Yani, toplam çarpımdan büyük olur. O
halde, 1’den büyük olan sayıların sayısı en fazla 6 olabilir.
Alıştırma : 100 tane pozitif tek sayının toplamı çarpımından büyüktür. Bu sayılar­
dan en fazla kaçı 1’den büyük olabilir?
Örnek 4
105 110 115 120 125
,
,
,
,
, ... rasyonel sayılarından kaçı tamsayıdır?
4
5
6
7
8
Çözüm : Verilen rasyonel sayıların payı 5’er 5’er artmakta, paydası ise 1’er 1’er
artmaktadır. Dikkat edilirse sayıların herbiri,  = 1 2 3  olmak üzere,
100 + 5
+3
formundadır ve bu şekilde devam etmektedir. Bu kesiri düzenlersek,
5 ( + 3) + 85
5 ( + 3)
85
100 + 5
=
=
+
+3
+3
+3
+3
85
5 · 17
= 5+
=5+
+3
+3
elde edilir. Bu ifadenin bir tamsayı olabilmesi için,  + 3 sayısı 85’e bölünmelidir.
Buna göre,
 + 3 ∈ {5 17 85}
olabilir. Yani,  = 2  = 14 ve  = 82 için tamsayı elde ederiz.
Örnek 5 n2 +n3 sayısı tamkare olacak şekilde 100’den küçük kaç n pozitif tam­
sayısı vardır?
Çözüm : 2 + 3 = 2 ( + 1) sayısının tamkare olabilmesi için,  + 1 sayısı bir
tamkare olmalıdır. Yani,  ∈ Z+ için,
 = 2 − 1 ve  ≤ 100
olmalıdır. O halde,  ∈ {2 3  10} için 2 + 3 sayısı tamkare olur ve 9 tanedir.
14
Matematik Olimpiyatlarına Hazırlık 1
Örnek 6 a, b, c, d ve e birbirinden farklı birer rakam ve " : " işareti bölme
işlemini göstermek üzere, a : b : c : d : e işleminde parantezler kullanılarak elde
edilebilecek en büyük sayı kaçtır?
Çözüm : Herhangi,  :  :  : · · · :  ifadesinde,  ve  hariç diğer tüm sayıları
parantezler kullanarak pay veya paydadan istediğimiz yere koymamız mümkündür.
 kesinlikle kesirin “payın”da,  ise “payda”sındadır. Buna göre,  :  :  :  : 
ifadesinin, en büyük olması için,
 = 9  = 8  = 7,  = 6 ve  = 1
alınabilir. Bu durumda sayımız, en büyük
9·8·7·6
= 3024
1
olabilir ve parantezlerle 9 {[((18) 7)] 6} şeklinde gösterebiliriz.
Örnek 7 10’dan küçük olan ve en sadeleşmiş durumda paydası 30 olan tüm pozi­
tif rasyonel sayıların toplamını bulunuz. (AIME 1992)
Çözüm : 0 ile 1 arasında istenen şekildeki rasyonel sayılar
1 7 11 13 17 19 23 29
      
30 30 30 30 30 30 30 30
sayılarıdır. Bu sayıların toplamı ise
1 + 7 + 11 + 13 + 17 + 19 + 23 + 29
=4
30
olur. 1 ve 2 arasındaki istenen şekildeki sayıları, bu sayılara 1 ekleyerek elde ede­
biliriz. 2 ve 3 arasındaki istenen sayılar için, bu sayılara 2 ekleyerek elde edilebilir.
Benzer şekilde düşünerek, istenen toplam
9 · 10
10 · 4 + 8 (1 + 2 + · · · + 9) = 40 + 8 ·
= 400
2
bulunur.
Örnek 8 Rakamları birbirinden farklı 9 basamaklı bir sayının herhangi yedi raka­
mı silindiğinde elde edilen iki basamaklı sayıya özsayı diyelim. Özsayıların sadece
birinin asal olabilmesi için, 9 basamaklı sayıda hangi rakam kullanılmamalıdır?
Çözüm : 0, 5 ve çift rakamları sayının en sonunda kullanalım. Bu sayılar özsayının
sonunda kaldığında özsayı asal olamaz. Örneğin, ...246850 yazabiliriz. Geriye 1,
3, 7 ve 9 rakamları kaldı. 7 rakamını 3, 1, 9 ile kullandığımızda daima asal sayı elde
edeceğimizden 7 sayısını kullanmamalıyız. 91 ve 93 sayıları da asal olmadığından,
913246850
sayısı istenen şekilde sayıdır ve burada sadece 13 sayısı asal olabilir.
Temel Bilgiler
15
1
Örnek 9 a, b ve c birbirinden farklı rakamları göstermek üzere,
= 0, abc
n
olacak şekilde kaç n pozitif tamsayısı vardır?
1
Çözüm : = 0  eşitliğinden,

1

999
33 · 37
=
veya  =
=

999


olmalıdır. Buna göre,  üç farklı rakamdan oluşacak şekilde, 33 · 37 sayısının
çarpanlarını incelersek, istenen şekildeki sayılar sadece 037 ya da 027 olabilir. (Diğer
çarpanlar üç farklı rakamı içermezler. 1, 3, 9, 111, 333, 999.) O halde,
999
999
=
= 27 ve  =
= 37
37
27
olduğundan, verilen eşitliği sağlayan iki değişik  sayısı vardır.
Örnek 10 Toplamları 500 olan pozitif tamsayıların çarpımı en büyük kaç ola­
bilir?
Çözüm : En büyük çarpanı aradığımız için, toplanan sayılar 4’ten büyük olamaz.
Çünkü, 4’ten büyük herhangi bir  sayısı için, 3 ( − 3) çarpımı daha büyüktür.
Örneğin, 5 yerine, 3 + 2 yazılırsa, 3 · 2 = 6 olduğundan, daha büyük bir çarpım
elde etmemizi sağlar. Toplananlar arasında 1 sayısı da olamaz. Çarpımı değiştirmez.
Toplanan sayılar arasında 4 yerine, 2 tane 2 kullanabiliriz. Bu durumda çarpım değiş­
meyecektir.
Sonuç olarak, toplamları sabit olan sayıların çarpımının en büyük olması için,
toplanan sayıları 3 ve 2 sayılarından oluşturmak gerekir. Fakat, 2’den fazla 2 kullan­
mak yerine, 3 kullanmak daha büyük sayıya ulaşmamızı sağlayacaktır. Bunu bir kaç
örnekle görmek mümkündür. Örneğin, 6 = 2 + 2 + 2 için, çarpım 8 elde edilirken,
6 = 3 + 3 için çarpım 9 elde edilir.
O halde, toplananlar arasında en fazla 2 tane 2 olmalıdır. 500 = 166 · 3 + 2
olduğundan, toplanan sayıların 166 tanesi 3 ve 1 tanesi 2 olmalıdır. Bu durumda en
büyük çarpım : 3166 2 olur.
Soru : Siz de, toplamları 301 olan sayıların çarpımlarının en büyük 4 · 399 olabile­
ceğini görünüz.
16
Matematik Olimpiyatlarına Hazırlık 1
1.2
Ardışık Sayıların Toplamı
I Bir sayı dizisinin ardışık terimlerine, dizinin ardışık sayıları denir.  bir tamsayı
olmak üzere, ardışık üç tamsayıyı  +1 +2 ve ardışık üç çift sayıyı da 2 2+2
2+4 şeklinde yazarız. En genel halde, bir  tamsayısının katı olan ardışık üç tamsayı
sırasıyla; ,  +  ve  + 2 biçiminde yazılır. Hep aynı değer artarak (veya
azalarak) elde edilen ardışık sayılara, aritmetik olarak artan (veya azalan) sayılar
denir.
Her defasında  artan, ’den başlayıp ’de biten ardışık sayıların sayısı,
−
+1

formülüyle bulunur. Örneğin, 5 12 19 26 96 sayı dizisinde
96 − 5
+ 1 = 14
7
sayı vardır. Ardışık sayıların toplamı da,
µ
¶
−
+
·
+1
 + ( + ) + ( + 2) + · · · +  =
2

ile bulunur. Yani terim sayısı, ilk ve son terimin ortalamasıyla çarpılır. Örneğin
96 + 5
5 + 12 + 19 + 26 + · · · + 96 = 14 ·
= 707
2
bulunur. Buna göre, ardışık ilk  sayının toplamı,
 ( + 1)
1 + 2 + 3 +··· +  =
2
ile ve ardışık ilk  tek sayının toplamı da
1 + 3 + 5 + · · · + (2 − 1) = 2
formülüyle hesaplanabilir. Örneğin,
10 + 11 + 12 + · · · + 20 = (1 + 2 + 3 + · · · + 20) − (1 + 2 + · · · + 9)
20 · 21 9 · 10
=
−
= 165
2
2
şeklinde hesaplanabilir.
Örnek 11 1’den 1071’e kadar olan tüm tamsayıları bir sırada
12345678910111210701071
şeklinde yazarak yeni bir sayı elde ediliyor.
a) Bu sayı kaç basamaklıdır?
b) Bu sayının 1000’inci rakamı kaçtır?
c) Bu sayının rakamları toplamı kaçtır?
Temel Bilgiler
17
Çözüm : a) Basamak sayısına göre kaçar rakam kullanıldığını hesaplayalım.
Bir basamaklı 1 2  9 sayılarında 9 rakam kullanılır.
İki basamaklı 10 11  99 sayılarının sayısı : 99 − 10 + 1 = 90’dır ve 90 · 2 = 180
rakam kullanılır.
Üç basamaklı 100 101 102  999 sayılarının sayısı : 999 − 100 + 1 = 900’dür ve
900 · 3 = 2700 rakam kullanılmıştır.
Dört basamaklı 1000 1001  1071 sayılarının sayısı da 1071 − 1000 + 1 = 72’dir
ve 72 · 4 = 288 rakam kullanılır. Böylece, toplam 9 + 180 + 2700 + 288 = 3177
rakam kullanılır. Yani, sayı 3177 basamaklıdır.
b) 9 tane bir basamaklı sayı ve 90 tane de iki basamaklı sayı vardır. O halde, bir ve
iki basamaklı sayıların kullanılmasıyla
9 + 90 · 2 = 189
rakam kullanılır. O halde, 1000−189 = 811 rakam üç basamaklı sayılardan olacaktır.
811 = 3 · 270 + 1
olduğundan üç basamaklı 270 sayı kullanılmış ve 271’inci sayının ilk rakamı bize
istediğimiz sayıyı verecektir. Üç basamaklı sayılar, 100’den başladığından 270’inci
üç basamaklı sayı, 100 + 270 − 1 = 369’dur. Böylece, 371’inci üç basamaklı sayı
370 ve ilk rakamı 3 olduğundan, oluşturulan sayının 1000’inci rakamı 3’tür.
c) Önce, 1 2 3  999 sayılarının rakamlarıyla oluşturulan sayının rakamları topla­
mını bulalım. 0 1 2  9 rakamları ile oluşturulacak üç basamaklı bir sayıda her
bir basamakta 0 1 2  9 rakamları 100010 kez bulunur. O halde 1 2 3  999
sayılarının rakamlarının toplamı
µ
¶
1000
(0 + 1 + · · · + 9) ·
· 3 = 13500
10
olur. O halde, geriye 1000 1001 1002  1071 sayılarının rakamları toplamını hesap­
lamak kaldı. Binler basamağındaki birlerin toplamı 72’dir. Yüzler basamağındaki
sayıların toplamı 0’dır. Onlar basamağında ise, 1,2,3,4,5,6 onar kez, 7 ise iki kez
kullanıldığından, bunların toplamı :
(1 + 2 + 3 + 4 + 5 + 6) 10 + 14 = 224
bulunur. Birler basamağında ise, 2 3  9 rakamları 7’şer, 1 rakamı ise 8 defa kul­
lanıldığından, rakamların toplamı
9 · 10
· 7 + 1 = 316
(1 + 2 + 3 + · · · + 9) 7 + 1 =
2
olur. Sonuç olarak, 12345678910111210701071 sayısının rakamları toplamı :
13500 + 224 + 316 = 14 040
elde edilir.
18
Matematik Olimpiyatlarına Hazırlık 1
Alıştırma :
a) 1’den 100’e kadar (1 ve 100 dahil) sayıların yanyana yazılmasıyla elde edilen
sayının rakamları toplamını bulunuz.
b) 1’den 2013’e (1 ve 2013 dahil) kadar sayıların yanyana yazılmasıyla elde edilen
sayının rakamları toplamını bulunuz.
Yanıt :
a) 10 (1 + 2 + 3 + · · · + 9) · 2 + 1 = 901
b) 13500 + 14500 + 83 = 28 083
Örnek 12 2000’den küçük pozitif tamsayıların herbirinin rakamları toplamı he­
saplanıyor ve aynı toplamı veren sayılar arasında daima en küçük olan sayı alı­
narak bir küme oluşturuluyor.
a) Bu kümenin eleman sayısı kaçtır?
b) Bu kümedeki tüm sayıların toplamı kaçtır?
c) Bu küme 222 elemanlı olsaydı, sayısal değeri en büyük eleman en az kaç olurdu?
Çözüm :
a) Bu kümedeki, rakamları toplamı en büyük olan eleman 1999 olacaktır ki, bu sayının
rakamları toplamı da 28’dir. Buna göre, rakamları toplamı 1 olan, 2 olan, ....,28 olan
en küçük elemanları sırasıyla yazarak, kümenin,
 = {1 2  8 9 19 29 39  99 199 299  999 1999}
şeklinde olacağı hemen görülebilir. Rakamları toplamı 1,2,3,...,28 değerlerini alabilir.
Yani, ’nin eleman sayısı 28’dir.
b) Kümedeki 9’arlı elemanların toplamı,
9 · 10
1 + 2 + ··· + 9 =
= 45
2µ
¶µ
¶
99 + 19
99 − 19
+1
= 531
19 + 29 + · · · + 99 =
10
2
µ
¶µ
¶
999 − 199
999 + 199
199 + 299 + · · · + 999 =
+1
= 5391
100
2
olduğundan, istenen toplam 45 + 531 + 5391 + 1999 = 7966 olur.
c) Küme 222 elemanlı olsaydı, en büyük elemanın rakamları toplamı 222 olurdu.
222 = 9 · 24 + 6 olduğundan, en büyük eleman, 24 tane 9 olmak üzere 69999 sayısı
olurdu.
19
Temel Bilgiler
1.3
Bölünebilme Kuralları
1) 2’ye bölünebilme : Sayı çift ise 2’ye bölünür.
2) 3’e bölünebilme : Sayının rakamları toplamı 3’ün katı ise 3’e bölünür.
3) 4’e bölünebilme : Sayının son iki rakamı 4’e bölünürse sayı 4’e bölünür.
4) 5’e bölünebilme : Sayının son rakamı 0 veya 5 ise sayı 5’e bölünür.
5) 6’ya bölünebilme . Sayı hem hem 2 hem de 3’e bölünürse 6’ya bölünebilir.
6) 7’ye bölünebilme : 10 9 8 7 6 5 4 3 2 1  10 basamaklı sayısını göz önüne
alalım. Bu sayının birler basamağından itibaren rakamlarını sırasıyla 1 3 ve 2 ile
çarparız. Daha sonra her bir üçlüyü toplarız. Birinci üçlü + ile ikinci üçlü − ile
üçüncü üçlü + ile çarpılır ve bulunan sayılar toplanır. Elde edilen sayı 7’ye bölünürse
sayı 7’ye bölünür.
(1 + 32 + 23 ) − (4 + 35 + 26 ) + (7 + 38 + 29 ) − (10 )
Örneğin 4292 736 sayısı için (6 + 3 · 3 + 2 · 7) − (2 + 3 · 9 + 2 · 2) + 4 = 0 oldu­
ğundan sayımız 7’ye bölünür.
7) 8’e bölünebilme : Sayının son üç rakamı 8’e bölünüyorsa sayı 8’e bölünür.
8) 9’a bölünebilme : Sayının rakamları toplamı 9’a bölünüyorsa sayı 9’a bölünür
9) 10’a bölünebilme : Sayının son rakamı 0 olmalıdır.
10) 11’e bölünebilme : Sayının tek numaralı basamaktaki sayıların toplamı ile
çift numaralı basamaklardaki sayıların toplamının farkı 11 ( ∈ Z) şeklinde ise
sayı 11’e bölünür.
Bir sayının, herhangi iki sayının çarpımına bölünebilmesi için, bu sayıların her
birine bölünebilmesi gerekir. Örneğin, bir sayının 15’e bölünebilmesi için hem 3’e
hem de 5’e, 45’e bölünebilmesi için de, hem 5’e ve hem de 9’a tam bölünebilmelidir.
101 tane
z }| {
Örnek 13 101, 1001, 10001, 100001, ...,1 00...00 1 sayılarından kaç tanesi
11’e bölünebilir?
Çözüm : 11’e bölünebilme kuralına göre tek sayıda 0 olursa sayı 11’e bölünemeye­
cektir. Yani 11’e bölünebilmesi her sayıda çift sayıda 0 olması gerekir. Buna göre
2 4 6... 100 sıfır olan sayıların sayısı : (100 − 2) 2 + 1 = 50 tane olur.
Örnek 14 A sayısının rakamları toplamı 29, B sayısının rakamları toplamı ise
17’dir. Buna göre, A · B çarpımının 400400 olamayacağını gösteriniz.
Çözüm :  sayısının 3’e bölümünden kalan 2 ve  sayısının 3’e bölümünden kalan
2’dir. O halde,  ·  sayısının 3’e bölümünden kalan 1 olmalıdır. Fakat, 400400
sayısının 3’e bölümünde kalan 2 olduğundan,  ·  çarpımı 400400 olamaz.
20
Matematik Olimpiyatlarına Hazırlık 1
Örnek 15 Beş basamaklı a679b sayısının, 72’ye bölünebilmesi için, a + b kaç
olmalıdır? (KANADA M.O 1980)
Çözüm : 679 sayısı hem 8 hem de 9’a bölünebilmelidir. 8’e bölünme kuralından
son üç basamak 8’in katı olması gerektiğinden ve 99 · 8 = 792 olduğundan  = 2
olmalıdır. 9’a bölünebilme kuralına göre rakamları toplamı
 + 6 + 7 + 9 + 2 =  + 24
sayısı 9’un katı olmalı yani  = 3 olmalıdır. O halde  +  = 5 bulunur.
Not :  ·  sayısının bir  sayısına bölümünden kalan,  ve  sayılarının  sayısına
bölümünden elde edilen kalanların çarpımının ’ye bölünmesiyle elde edilen kalana
eşittir.
101 tane
z }| {
Örnek 16 101 · 1001 · 10001 · · · 1 00...00 1 çarpımının 9’a bölümünden kalan
kaçtır?
Çözüm : Çarpımdaki her bir çarpanın 9’a bölümünen kalan 2’dir. 101 tane çarpan
olduğundan, verilen çarpımın 9’a bölümünden kalan
2 · 2 · · · 2 = 2101
sayısının 9’a bölümünden kalana eşittir. 26 = 64’ün 9’a bölümünden kalan 1 olduğunu
kullanalım. 101 = 16 · 6 + 5 olduğundan,
2101 = 26 · 26 · · · 26 · 25
sayısının 9’a bölümünden kalan 25 ’in 9’a bölümünden kalana eşit olacaktır. Bu ise,
5’tir.
Örnek 17 100’den küçük kaç n doğal sayısı için 4n2 +12n − 4 ifadesi 13’e
bölünür?
Çözüm : Bir sayı 13’e bölünüyorsa, 13 fazlası da bölünmelidir. Buna göre,
42 + 12 − 4 + 13 = 42 + 12 + 9 = (2 + 3)2
sayısı da 13’e bölünmelidir. Bunun için 2 + 3 sayısı 13’ün katı olmalıdır. 2 + 3
sayısı tek sayı olduğundan, 2 + 3 sayısı,
13 39 65 91 117 143 169 195
olabilir. Bu sayılardan sırasıyla,  sayısı
5 18 31 44 57 70 83 96
olabilir.
21
Temel Bilgiler
Örnek 18 Altı basamaklı aabbaa sayısı 7’ye bölünecek şekilde kaç (a, b) ikilisi
vardır?
Çözüm : 7’ye bölünebilme kuralına göre,
(1 + 3 + 2) − (1 + 3 + 2) =  − 
sayısının 7’nin katı olması gerekir. Buna göre,
 −  = 0 veya  −  = ±7
olabilir.  −  = 0 ise  = ’dir ve  =  ∈ {1 2 3 4 5 6 7 8 9} olabilir. Yani,
9 tane sayı 7’ye bölünür.  −  = ±7 ise, beş durum olabilir.  = 7  = 0  = 8
 = 1;  = 1  = 8 ve  = 9  = 2;  = 2  = 9 durumları. Buradan da, 770077
118811 881188 ve 229922 992299 sayıları da 7’ye bölünecektir. Sonuç olarak 14
tane ( ) ikilisi vardır.
Örnek 19 1212, 1313, 1414, ..., 4949 sayılarından kaç tanesi 13’e bölündü­
ğünde 10 kalanını verir?
Çözüm :  formundaki bir sayıyı :
 =  · 101
şeklinde yazabiliriz. Buna göre,  sayısının 13’e bölümünden kalanı bulmak için,
 sayısının 13’e bölümünden kalan sayı ile, 101 sayısının 13’e bölümünden kalan
sayıyı çarpmalıyız. Buna göre, 101 sayısının 13’e bölümünden kalan 10 olduğundan,
 sayısının 13’e bölümünden kalan 1 olması gerekir. Bu ise sadece  sayısının,
14 27 40
olması durumunda mümkündür. Yanıt 3.
Örnek 20 x,y,z,n ve m rakam olmak üzere, beş basamaklı xyz1n sayısıyla, üç
basamaklı 234 sayısının çarpımı 332m842 olduğuna göre, x+y+z+n+m =?
Çözüm : İkinci çarpanın rakamları toplamı 9’un katı olduğundan 9’a tam bölünür
dolayısıyla çarpım da bölünmelidir. Buna göre  = 5 olur.
3325842
= 14213
234
olduğundan  = 1  = 4  = 2 ve  = 3 olur. Böylece
 +  +  +  +  = 15
olur.
22
Matematik Olimpiyatlarına Hazırlık 1
Örnek 21 10’dan itibaren iki basamaklı bir sayının n kez yanyana yazılmasıyla
elde edilen, 2n basamaklı sayıyı a ile gösterelim. Buna göre, a1 =10, a2 = 1111,
a3 = 121212, 4 = 13131313 , ... şeklinde bir sayı dizisi oluşacaktır. Buna
göre, a1 +a2 +a3 + · · · + a50 toplamının 7’ye bölümünden kalan kaçtır?
Çözüm :  sayısını oluşturan iki basamaklı sayı ile ’nin farkı daima 9’dur.
1 = 10 ⇒ 10 − 1 = 9; 2 = 1111 ⇒ 11 − 2 = 9; 
Buna göre, 50’nci sayı, yani 50 sayısı, 59 sayılarından oluşacaktır. Diğer yandan, 6
basamaklı  formundaki bir sayı daima 7’ye bölünür. Gerçekten,
(1 + 3 + 2) − (1 + 3 + 2) = 0
olduğu görülebilir. Buna göre, 1 + 2 + 3 + · · · + 50 toplamının 7’ye bölümünden
kalan,
10 + 1111 + 13 + 1414 + 16 + 1717 + 19 + 2020 + · · · + 58 + 5959
toplamının 7’ye bölümünden kalana eşit olacaktır. Bu sayılar arasındaki 4 basamaklı
sayılar  formundadır ve
 =  · 101
yazalım. 101’in 7’ye bölümünden kalan 3’tür. O halde,  sayısının 7’ye bölümün­
den kalan, 3 ·  sayısının 7’ye bölümünden kalana eşit olacaktır. Böylece, istenen
kalan
(10 + 13 + 16 + · · · + 58) + 3 · (11 + 14 + 17 + 20 + · · · + 59)
toplamının 7’ye bölümünden kalan olacaktır.
µ
¶µ
¶
µ
¶µ
¶
58 − 10
58 + 10
59 − 11
59 + 11
+1
+3
+1
= 2363
3
2
3
2
sayısının 7’ye bölümünden kalan ise 4’tür.
Örnek 22 Rakamları birbirinden farklı m35n dört basamaklı sayısı, 36’ya bölün­
düğünde 23 kalanını verdiğine göre, m + n toplamı kaçtır?
Çözüm : Verilenlere göre,
35 = 36 + 23 = 4 (9 + 5) + 3 = 9 (4 + 2) + 5
şeklinde yazılabilir. Yani, 35 sayısı 4’e bölündüğünde 3 kalanını, 9’a bölündüğünde
de 5 kalanını vermelidir. 4’e bölündüğünde 3 kalanını verebilmesi  = 5 ve  = 9
iken mümkündür. Fakat rakamlar birbirinden farklı olduğundan  = 9 olmalıdır.
Buna göre,
359 iken, 9’a bölündüğünde 5 kalanını vermesi için  = 6;
olmalıdır. Böylece,  +  = 6 + 9 = 15 bulunur.
23
Temel Bilgiler
100 tane
z }| {
Örnek 23 S = 7 + 77 + 777 + 7777 + · · · +777...77 toplamının 9’a bölü­
münden kalan kaçtır?
Çözüm : İlk terimin 9’a bölümünden kalan 7’dir. Bundan sonraki her bir terimin
9’a bölümünde kalan, kendinden önce gelen terimin 9’a bölümünden kalandan 7 fazla
olacaktır. Buna göre, kalanlar, sırasıyla
7 5 3 1 8 6 4 2 0 7 5 
şeklinde devam edecektir. Yani, 9 farklı kalan elde edilecektir. 100 = 9 · 11 + 1
olduğundan, verilen sayının 9’a bölümünden kalan,
11 (0 + 1 + 2 + 3 + 4 + 5 + 6 + 7 + 8) + 7 = 403
sayısının 9’a bölümünden kalan eşittir. O halde, kalan 7 bulunur.
Örnek 24 234’e bölündüğünde 11 kalanını veren ve iki asal sayının toplamı veya
farkı şeklinde yazılan kaç tane pozitif tamsayı vardır?
Çözüm :  = 234 + 11 =  +  olsun. 234 + 11 tek sayı olduğu için  ve  asal
sayılarından biri tek biri çift olmalıdır. O halde, biri 2 olmalıdır.  = 2 alalım. Bu
durumda  = 234 + 9 = 9 (26 + 1) olur ki, bu sayı 9’a bölüneceğinden asal sayı
olamaz.
Şimdi ise  = 234 + 11 =  −  olsun. Benzer şekilde  = 2 olmak zorundadır.
Buna göre,
 = 234 + 13 = 234 + 13 = 13 (18 + 1)
olursa  = 0 ve  = 13 bulunur. Böylece , 11 bulunur. O halde istenen koşulları
sağlayan sadece bir pozitif sayı vardır, bu sayı da 11’dir.
Örnek 25 5’in katı olmayan herhangi n tamsayısının karesinin bir fazlasının 5’e
bölümünden elde edilebilecek kaç farklı kalan vardır?
Çözüm : 5’e bölünmeyen tamsayıları  = 5 + 1  = 5 + 2  = 5 + 3 veya
 = 5 +4 şeklinde yazabiliriz. olabilir. Hatta, 5 +4 yerine 5 −1 ve 5 +3 yerine
de 5 − 2 yazılarak, 5’e bölünmeyen tamsayıları kısaca,  = 5 ± 1 ve  = 5 ± 1
şeklinde gösterebiliriz.
Buna göre,
 = 5 ± 1 için, 2 + 1 = (5 ± 1)2 + 1 = 252 ± 10 + 2 = 5 + 2,
2
 = 5 ± 2 için, 2 + 1 = (5 ± 2) + 1 = 252 ± 20 + 5 = 5
olduğundan, sadece 0 ve 2 kalanları elde edilebilir.
24
Matematik Olimpiyatlarına Hazırlık 1
Örnek 26 n2 +n + 1 sayısının tüm rakamlarının toplamı 101 olacak şekilde kaç
tane n doğal sayısı vardır?
Çözüm : Bir  doğal sayısı için 2 ++1 sayısının 3’e bölündüğünde hangi kalanları
elde edebileceğimizi görelim.
 = 3 formunda olursa, 92 + 3 + 1 olacağından, 3’e bölümünden kalan 1 olur.
 = 3 + 1 formunda olursa,
¡
¢
(3 + 1)2 + (3 + 1) + 1 = 3 32 + 3 + 1
olduğundan kalan 0’dır.
 = 3 − 1 formunda olursa,
2
(3 − 1) + (3 − 1) + 1 = 92 − 3 + 1
olduğundan kalan 1’dir.
Yani, asla 2 kalanı elde edilemez. 2 +  + 1 sayısının 3’e bölümünden 2 kalanı
elde edilemiyorsa, rakamları toplamını 3’e böldüğümüzde de 2 kalanı elde edemeyiz.
Halbuki, 101 sayısının 3’e bölümünden kalan 2’dir. O halde, 2 +  + 1 sayısının
rakamları toplamı 101 olacak şekilde  doğal sayısı yoktur.
Örnek 27 1’den 100’e kadar (1 ve 100 dahil) tamsayılardan 51 tanesi seçiliyor.
Bu seçilen sayılardan bir diğerini bölecek şekilde iki sayının mutlaka bulunacağını
gösteriniz.
Çözüm : Herhangi bir pozitif tamsayıyı,  bir tek sayı olmak üzere, 2  formunda
yazabiliriz. 2’nin kuvvetlerinde  = 1 olacaktır. Seçilen sayılar 1’den 100’e kadar
olan sayılardan seçileceğinden,  sayısı 1 ile 100 arasında olacaktır. Yani,  sayısı
50 tane tek sayı değeri alabilir. Halbuki, biz 51 sayı seçiyoruz. O halde, seçilen
sayılardan ikisinin  değerleri aynı olacaktır.  değerleri aynı ise, 2  formundaki
bu iki sayının ’si küçük olan, ’si büyük olanı kesinlikle bölecektir.
(Bu soruda kullanılan yöntem Güvercin Yuvası ilkesi olarak bilinir. Bu ilke ikinci
ciltte detaylı incelenmiştir.)
25
Temel Bilgiler
1.4
Bir Sayının Pozitif Bölenlerinin Sayısı, Toplamı ve Çarpımı
F Herhangi bir  sayısını, 1  2    sayıları birbirinden farklı asal sayılar ve
1  2    sayıları da pozitif tamsayılar olmak üzere,

3
 = 11 · 2
2 · 3 · · · 
formunda yazabiliriz. Bu yazılışa,  sayısının asal çarpanlarına göre yazılışı denir.
F Pozitif Bölen Sayısı
Bir  sayısının asal çarpanları ile yazılımı

3
 = 11 · 2
2 · 3 · · · 
olsun. Bu durumda  sayısının pozitif bölenlerinin sayısı
 () = (1 + 1) (2 + 1) (3 + 1) · · · ( + 1)
formülüyle bulunur. Bir  sayısının pozitif bölenlerinin tek sayı olması için,  sayısının
tüm asal çarpanlarının kuvvetlerinin çift sayı olması gerekir. Kısaca,  sayısı tamkare
olmalıdır. Örneğin,  = 24 32 56 sayısının pozitif bölenlerinin sayısı tektir.  sayısının
tüm (hem negatif hem de pozitif) bölenlerinin sayısı ise 2 ·  ()’dir.
F Pozitif Bölenlerin Toplamı
 sayısının pozitif bölenlerinin toplamı ise
 () =
 +1 − 1
11 +1 − 1 22 +1 − 1 13 +1 − 1
··· 
1 − 1
2 − 1
3 − 1
 − 1
formülüyle bulunur.
F Pozitif Bölenlerin Çarpımı

sayısı da bir pozitif bölendir.
Bir  sayısı, bir  sayısının pozitif böleni ise,

15
(Örneğin, 3, 15’in pozitif böleni iken,
= 5 de pozitif bölenidir.) Bu özellik, bize
3

tüm pozitif bölenlerin çarpımını kolayca hesaplayabilmemizi sağlar. Çünkü,  ve

bölenlerin çarpımı, ’dir.
* Sayı tamkare değilse, bölenlerin çarpımı bölen sayısının yarısı kadar ’nin
kendisiyle çarpılmasıyla elde edilir.
Yani, ç() = ()2 olur.
Örneğin, 12 sayısının pozitif bölenlerinin çarpımı 1 · 2 · 3 · 4 · 6 · 12 = 123 ’dür. Yani,
ç(12) = 1262 ’dir.3
* Sayı tamkare ise, bölenlerin çarpımı, bölen sayısının 1 eksiğinin yarısı kadar
’nin kendisiyle
çarpımıyla ’nin karekökünün çarpımına eşittir. Yani, ç() =
√
(()−1)2  ’dir.
Örneğin, 16 sayısının pozitif bölenlerinin çarpımı da,
1 · 2 · 4 · 8 · 16 = 162 · 4 = 1024
√
dür. Yani, ç(16) = 16(5−1)2 16 = 1024’tür.
26
Matematik Olimpiyatlarına Hazırlık 1
Örnek 28 48 sayısının pozitif bölenlerinin sayısını, toplamını ve çarpımını bu­
lunuz.
Çözüm : 48 = 24 3 olduğundan, pozitif bölen sayısı
 (48) = (4 + 1) (1 + 1) = 10
pozitif bölenlerinin toplamı,
25 − 1 32 − 1
·
= 124
2−1 3−1
pozitif bölenlerinin çarpımı ise, bölen sayısı 10 olduğundan,
 (48) =
ç(48) = 48102 = 485
olur.
Örnek 29 3600 sayısının pozitif bölenlerinden çift olanlarının sayısını bulunuz.
Pozitif çift tamsayı bölenlerinin toplamını bulunuz.
Çözüm : 3600 = 24 52 32 olduğundan, toplam 5 · 3 · 3 = 45 pozitif böleni vardır. Tek
olanlar ise, 5 3 formundadır. Bunların sayısı da, 3 · 3 = 9 olduğundan, çift olanların
sayısı 45 − 9 = 36 olur.
Pozitif çift tamsayı bölenlerinin toplamı ise,
¡
¢¡
¢¡
¢
2 + 22 + 23 + 24 1 + 5 + 52 1 + 3 + 32 = 30 · 31 · 13
çarpımına eşittir. Buradan, 12 090 bulunur.
Örnek 30 Pozitif bölenlerinin sayısı tek sayı olan,2013’ten büyük en küçük tam­
sayı kaçtır?
Çözüm : Sayının bölen sayısının tek sayı olması için, sayı tamkare olmalıdır. Buna
göre, 452 = 2025 olduğundan istenen şekildeki en küçük tamsayı 2025’tir.
Örnek 31 1013’i böldüğünde 13 kalanı elde edilen kaç tane pozitif çift tamsayı
vardır?
Çözüm : 1013 =  ·  + 13 ve   13 yazılabilir. Buradan  ·  = 1000 = 23 53
eşitliğine göre
(3 + 1) (3 + 1) = 16
tane pozitif bölen vardır. Bu bölenler arasından 13 kalanından büyük olmayanları
çıkarmalıyız. Çünkü, bu sayılar, bir sayıyı böldüğünde 13 kalanını veremez. 1 2 5
10 çıkarılırsa 16 − 4 = 12 tane istenen şekilde  pozitif tamsayısı vardır. Fakat,
soruda pozitif çift sayı denildiğinden, 125 çarpanını çıkarmalıyız. Buna göre, 1013’ü
böldüğünde 13 kalanı elde edilen 12 − 1 = 11 çift sayı vardır.
27
Temel Bilgiler
Örnek 32 48 tane pozitif böleni olan en küçük pozitif tamsayı kaçtır?
Çözüm : En küçük sayıyı bulabilmek için, asal çarpanları mümkün olduğu kadar
küçük asallardan seçeriz. Diğer taraftan seçtiğimiz bu asal sayıların üslerinin 1 faz­
lalarının çarpımları 48 olmalıdır.
 = 2 3 5 7
olduğunu farzedelim. 48 = 16·3 olduğundan, üslerden biri mutlaka 2 olmalıdır. Buna
göre,  = 3  = 2 ve  =  = 1 alınırsa,  = 23 ·32 ·5·7 = 2520 bulunur. Bu sayının
pozitif bölenlerinin sayısı gerçekten, (3 + 1) (2 + 1) (1 + 1) (1 + 1) = 48’dir.
Örnek 33 2 ve 3’e bölünen pozitif bir tamsayının tam 33 pozitif böleni varsa, bu
sayı en küçük kaç olabilir?
Çözüm : Pozitif bölenlerinin sayısı 33 ve 33 = 3 · 11 olduğundan istenen sayının 2
ve 3’ten başka asal böleni olamaz. Buna göre, istenen şekildeki en küçük sayı 210 · 32
elde edilir.
Örnek 34 108 000 sayısının pozitif bölenlerinin kaç tanesi 8’e bölünür ama 9’a
bölünmez?
Çözüm : 108000 = 25 33 53 biçiminde asal çarpanlarına ayrılabilir. Buna göre, iste­
diğimiz pozitif bölenler, 2 3 5 formundaki sayılar arasından 3 ≤  ≤ 5 0 ≤  ≤ 1
ve 0 ≤  ≤ 3 koşulunu sağlayan sayılardır. Dolayısıyla, istenen şekildeki sayıların
sayısı : 3 · 2 · 4 = 24 olarak bulunur.
n (n + 1) (2n + 1)
olduğuna göre,
6
12 −1 + 22 −2 + 32 −3 + · · · + 1002 −100
Örnek 35 12 +22 +32 + · · · + n2 =
ifadesinin kaç tane tek sayı pozitif böleni vardır?
Çözüm : Verilen sayıyı,
¢
¡ 2
1 + 22 + · · · + 1002 − (1 + 2 + · · · + 100) =
100 · 101 · 201 100 · 101
−
6
2
= 50 · 101 (67 − 1)
= 22 · 3 · 52 · 11 · 101
biçiminde hesaplayabiliriz. Buna göre, tek sayı pozitif bölenlerinin sayısı,
(1 + 1) (2 + 1) (1 + 1) (1 + 1) = 24
olarak bulunur.
28
Matematik Olimpiyatlarına Hazırlık 1
Örnek 36 992 +1, 992 +2, ..., 1042 −2, 1042 −1, 1042 sayılarından 99’a bölü­
nenlerin toplamının, kaç pozitif böleni vardır?
Çözüm : Çözümümüzü en genel halde yapalım. Herhangi,
2 + 1 2 + 2  ( + 5)2
sayı dizisinde, Terim Sayısı = Son Terim − İlk Terim +1 olduğu kullanılarak,
¡
¢
( + 5)2 − 2 + 1 + 1 = 10 + 25
sayı olduğu görülebilir. Buna göre,   25 için, verilen sayıların arasından  ile
bölünenler sadece
2 +  2 + 2 2 + 3  2 + 10
sayılarıdır. Bunların toplamı da,
¡ 2
¢ ¡
¢
¡
¢
 +  + 2 + 2 + · · · + 2 + 10 = 55 + 102 = 5 (2 + 11)
elde edilir. Böylece,  = 99 için, toplam
5 (2 + 11) = 5 · 99 · (2 · 99 + 11) = 32 · 5 · 112 · 19
elde edilir. Buna göre, pozitif bölenlerinin sayısı :
(2 + 1) (1 + 1) (2 + 1) (1 + 1) = 36
bulunur.
29
Temel Bilgiler
1.5
EBOB - EKOK
I İki veya daha fazla sayının her birini bölebilen en büyük pozitif tamsayıya bu
sayıların ortak bölenlerinin en büyüğü denir.   iki tamsayı olmak üzere  ve 
sayılarının ortak bölenlerinin en büyüğü EBOB ( ) veya bazen kısaca ( ) ile
gösterilir. EBOB ( ) = 1 ise  ve  sayılarına aralarında asal sayılar denir.
Ardışık iki pozitif sayı aralarında asaldır.
Örnek 37 Kareleri 2 ile 200 sayıları arasında olan, herhangi ikisi aralarında asal
olacak şekilde en fazla kaç sayı seçilebilir?
Çözüm : 142 = 196 olduğundan, kareleri 2 ile 200 arasında olan sayılar 2, 3, 4, 5, 6,
7, 8, 9, 10, 11, 12, 13, 14’tür Bu sayıların arasından herhangi ikisinin aralarında asal
olması için,
2 3 5 7 11 13
asal sayılarını seçebiliriz. Eğer 7 sayı seçilirse iki sayının asal çarpanları arasında bu
asal sayılardan biri ortak olacaktır ki bu da aralarında asal olma koşulunu bozacaktır.
O halde en fazla 6 sayı seçilebilir.
İki sayının EBOB’unu bulmak için, sayılar asal çarpanlarına ayrılır ve her iki
sayıda da bulunan ortak asal çarpanlardan, üssü küçük olanlar çarpılır.
Örneğin,
¡
¢
EBOB 24 32 52  71 22 33 = 22 32 = 36
şeklinde bulunur. Yani, ortak asal çarpanlardan, üssü en küçük olanları çarptık.
Örnek 38 4050 ’nin böleni olup 5040 ’ın böleni olmayan kaç pozitif tamsayı vardır?
Çözüm : 4050 = 2150 550 olduğundan 4050 sayısının, (150 + 1) (50 + 1) tane pozitif
böleni vardır. Bu bölenlerden, 5040 ’ın bölenleri olanları çıkaralım. Bunun için, her
ikisinin bölenlerinin sayısını bulmalıyız. 5040 = 240 580 ve
¡
¢
EBOB 5040  4050 = 240 550
olduğundan, her ikisinin de böleni olan (40 + 1) (50 + 1) pozitif tamsayı vardır. O
halde, 4050 ’nin böleni olup 5040 ’ın böleni olmayan,
pozitif tamsayı vardır.
151 · 51 − 41 · 51 = 51 · 110 = 5610
30
Matematik Olimpiyatlarına Hazırlık 1
Örnek 39 504 veya 203 sayılarını bölen kaç pozitif tamsayı vardır?
Çözüm : Sorudaki veya bağlacı çok önemli bir ayrıntıdır. 504 = 24 58 sayısının
pozitif bölenlerinin sayısı 5 · 9 = 45 203 = 26 53 sayısının pozitif bölenlerinin sayısı
ise, 7 · 4 = 28’dir. Fakat, cevap 45 + 28 = 73 değildir. Çünkü, 73 sayısının içinde,
504 ve 203 sayılarının her ikisini de bölen sayılar, iki kez hesaplanmıştır. O halde, 504
ve 203 sayılarının her ikisini de bölen sayıların sayısını çıkarmalıyız. Her ikisini de
bölen sayılar, aslında EBOB’unu bölen sayılardır.
¡
¢
¡
¢
EBOB 504  203 =EBOB 24 58  26 53 = 24 53
olduğundan, her ikisini de bölenlerin sayısı (4 + 1) (3 + 1) = 20 olur. O halde,
istenen yanıt : 73 − 20 = 53 bulunur.
Örnek 40 EBOB’ları 48, toplamları ise 576 olacak şekilde kaç pozitif tamsayı
ikilisi vardır?
Çözüm : Bu iki sayıya  ve  diyelim.  ve  aralarında asal olmak üzere,  = 48
ve  = 48 biçiminde yazılabilir. Toplamları 576 ise,  +  = 48 ( + ) = 576
eşitliğinden,  +  = 12 olacaktır.  ve  aralarında asal olacağından (1 11) veya
(5 7) ikililerinden biri olabilir. Bu durumda, bu sayılar 48 528 veya 240 336 ikilileri
olabilir.
F İki sayının ortak böleni, bu sayıların tamsayı katlarının farkını da böler. Bu
özelliği kullanarak iki sayının EBOB’unu bulmak mümkündür. (Öklid Algoritması)
Örnek 41 EBOB(10055, 22121) =?
Çözüm : Küçük olan sayının bir tamsayı katını büyük olan sayıdan çıkaracağız. Bu­
rada, 2 katını çıkarmamız yeterli. Buna göre, 22121 − 2 · 10055 = 2011 olduğundan,
EBOB(10055 22121) =EBOB(10055 2011)
bulunur. 10055 sayısı 2011 sayısının 5 katı olduğundan, her iki sayı da 2011’e bölü­
necektir. Yani, EBOB(10055 22121) = 2011 bulunur.
Örnek 42 n bir pozitif tamsayı olmak üzere, 3n+5 ve 7n+8 sayılarının EBOB’u
kaç farklı pozitif tamsayı olabilir?
Çözüm : Bu iki sayının EBOB’u 3+5 ile (7 + 8)−2 (3 + 5) = −2 sayılarının
EBOB’una eşittir. O halde, 3+5 ile −2 sayılarının EBOB’unu bulmamız yeterlidir.
Bu iki sayının EBOB’u ise  − 2 sayısı ile 3 + 5 − 3 ( − 2) = 11 sayılarının
EBOB’una eşittir. Dolayısıyla 3 + 5 ve 7 + 8 sayılarının EBOB’u ya 1 veya 11
olabilir. EBOB’un 11 olması için,  − 2 sayısının 11’e bölünmesi gerekir. Örneğin,
 = 13 ise sayılar, 3 · 13 + 5 = 44 ve 7 · 13 + 8 = 99 olur ki, bunların EBOB’u 11’dir.
31
Temel Bilgiler
Örnek 43 n sayısı 2011’den büyük bir tamsayı olduğuna göre, 11n+9 ve 3n+2
aralarında asal olmayacak şekilde en küçük  sayısını bulunuz.
Çözüm : Bu iki sayının EBOB’u 3 + 2 ile 11 + 9 − 3 (3 + 2) = 2 + 3 sayısının
EBOB’una eşittir. Bunların EBOB’u da 2 + 3 ile 3 + 2 − (2 + 3) =  − 1
sayılarının EBOB’una eşittir. O halde, EBOB( − 1 2 + 3)’ü bulmalıyız.
EBOB( − 1 2 + 3) =EBOB( − 1 2 + 3 − 2 ( − 1)) =EBOB( − 1 5)
olduğundan 5 olabilir. Bunun için,  − 1 sayısı 5’in katı olmalıdır.  sayısı 2011’den
büyük olduğundan, en küçük  sayısı için,  − 1 = 2015 ve  = 2016 alınabilir.
2n − 3
kesiri, 1’den büyük bir n tamsayısı için, a pozitif sayısı ile
5n − 1
sadeleştirilebildiğine göre, a sayısı kaçtır?
2 − 3
Çözüm :
kesiri  sayısı ile sadeleştirilebildğine göre,  sayısı, 2 − 3 ve
5 − 1
5 − 1 sayılarının her ikisinin de bölenidir. Buna göre,  sayısı,
Örnek 44
(5 − 1) − 2 (2 − 3) =  + 5
sayısını da bölmelidir. Benzer düşünce ile,  sayısı, hem 2 −3 hem de,  +5 sayısını
bölüyorsa,
2 ( + 5) − (2 − 3) = 13
sayısını da bölmelidir. O halde,  sayısı 13’tür.
I İki veya daha fazla sayının her birine bölünen en küçük pozitif tamsayıya bu
sayıların ortak katlarının en küçüğü denir.   iki tamsayı olmak üzere  ve 
sayılarının ortak katlarının en küçüğü EKOK( ) veya kısaca [ ] ile gösterilir.
İki sayının EKOK’unu bulmak için, sayılar asal çarpanlarına ayrılır ve sayılardaki asal çarpanlardan üssü en büyük olanlar çarpılarak EKOK bulunur.
Örneğin,
¢
¡
EKOK 24 32 52  71 22 33 = 24 33 52 7 = 75 600
şeklinde bulunur. Yani, her asal çarpandan, üssü en büyük olanları çarptık.
Örnek 45 391 sayısı 12 tane pozitif tamsayının toplamı olduğuna göre bu 12
sayının EKOK’u en küçük kaç olabilir?
Çözüm : 12 · 33 = 396 olduğundanEKOK’un 33 ve 33’ten küçük olması mümkün
değildir. Buna göre EKOK acaba 34 olabilir mi? EKOK = 34 olması için sayılar
34 17, 2 ve 1’den oluşmalıdır. 11 tane 34 ve 1 tane 17 alınırsa, 34 · 11 + 17 = 391
olacağından, EKOK en küçük 34 olabilir.
32
Matematik Olimpiyatlarına Hazırlık 1
I İki sayının EKOK’ı ile EBOB’larının çarpımı, bu iki sayının çarpımına eşittir.
Yani,
eşitliği sağlanır.
 ·  = EKOK ( ) · EBOB [ ]
Örnek 46 En küçük ortak katları 144 olan kaç farklı iki pozitif tamsayı buluna­
bilir? (Örneğin, bir tanesi (16, 18) ikilisidir. EKOK’ları 144’tür.)
Çözüm : Bu iki sayıyı  ve  ile gösterelim. 144 = 24 32 olduğundan,
 = 2 · 3 ve  = 2 · 3
formunda olacaktır. EKOK’un tanımı gereği,
 ve ’in en büyüğü 4,
 ve ’nin en büyüğü 2 olmalıdır.
O halde, bunları kaç farklı şekilde seçebileceğimizi görelim.
( ) ikilisi, (0 4) ; (1 4) ; (2 4) ; (3 4) ; (4 4) ; (4 3) ; (4 2) ; (4 1) ;
(4 0) olacak şekilde 9 farklı şekilde olabilir.
( ) ikilisi, (0 2) ; (1 2) ; (2 2); (2 1) ; (2 0) olacak şekilde 5 farklı şekilde
olabilir.
Sonuç olarak, bu iki gruptaki, her gruptan seçilen, her bir ikili için farklı iki sayı elde
edileceğinden, 9 · 5 = 45 farklı iki pozitif tamsayı bulunabilir.
Örneğin, ( ) = (4 2)  ( ) = (1 2) alınırsa,
 = 24 · 31 = 48 ve  = 22 · 32 = 36
elde edilir. EKOK(48 36) = 144’dür.
Alıştırma : Siz de, EKOK( ) = 23 32 52 olacak şekilde 175 tane ( ) pozitif
tamsayı ikilisi olduğunu görünüz.
Örnek 47 a ve b doğal sayılarının ortak bölenlerinin en büyüğü 9, ortak kat­
larının en küçüğü ise 270’dir. Buna göre, bu iki sayının toplamı en küçük kaç
olabilir? (Doğuş Ün. Fen Liseleri Y. 2005)
Çözüm : EBOB( ) · EKOK ( ) =  ·  olduğundan,
 ·  = 9 · 270 = 35 · 2 · 5
olur. Bu iki sayının olabilecek en küçük toplamını bulmak için,  ve ’yi birbirine
en yakın olacak şekilde seçmeliyiz. Her iki sayının da 9’a bölünebildiği de gözönüne
alınırsa,  = 9 · 5 ve  = 9 · 6 seçilebilir. Bu durumda en küçük toplam 45 + 54 = 99
olarak bulunur.
33
Temel Bilgiler
Örnek 48 En küçük ortak katları 120 ve en büyük ortak bölenleri 10 olan kaç
farklı sayı ikilisi vardır?
Çözüm : EBOB( ) · EKOK ( ) =  ·  olduğundan,
 ·  = 1200 = 2 · 2 · 3 · (10) · (10)
olmalıdır.  ve ’nin her ikisinde de 10 çarpanı bulunması gerekir. Buna göre, ( )
ikilisi, (20 60) ; (40 30)  (10 120) durumlarından biri olabilir. Yani, üç farklı sayı
ikilisi vardır.
Örnek 49 Üç ardışık sayıdan birincisi 3’e, ikincisi 8’e, üçüncüsü 13’e tam bölünü­
yor. Bu üç ardışık sayının toplamı en az kaçtır?
Çözüm : 3,8 ve 13 sayıları beşer beşer artmıştır. Buna göre, uygun bir  sayısıyla
toplayıp 5’e bölerek bu sayıları ardışık hale getirebiliriz.
3 +  8 +  13 + 


5
5
5
sayılarının, sırasıyla 3,8 ve 13’e bölünen birer tamsayı olması için,  sayısının 3, 8 ve
13’ye bölünen bir sayı olması gerekir. Buna göre,  en az,
EKOK (3 8 13) = 3 · 8 · 13 = 312
alınabilir. Böylece,
3 + 312
8 + 312
13 + 312
= 63
= 64 ve
= 65
5
5
5
sayıları, istenen koşulu sağlayan en küçük sayılar olacaktır. O halde bu üç ardışık
sayının toplamı en az 63 + 64 + 65 = 192 olarak bulunur.
Örnek 50 Birincisi 3’e, ikincisi 5’e, üçüncüsü 7’ye ve dördüncüsü 9’a tam bölü­
nen en küçük dört ardışık sayıyı bulunuz.
Çözüm : Bir önceki örnekte olduğu gibi
+3 +5 +7 +9



2
2
2
2
sayılarının istenen şekilde olması için
 = EKOK (3 5 7 9) = 5 · 7 · 9 = 315
alınmalıdır. Böylece sayılar
315 + 3 315 + 5 315 + 7 315 + 9



2
2
2
2
yani, 159 160 161 162 olarak bulunur.
34
Matematik Olimpiyatlarına Hazırlık 1
1.6
Modüler Aritmetik
 pozitif bir tamsayı olmak üzere  −  sayısı ’ye tam bölünebiliyor ise  ve 
sayıları modül ’ye göre denktirler denir ve
 ≡  (mod )
şeklinde yazılır. Yani  ≡  (mod ) ifadesinde   sayısının  ile bölümünden
elde edilen kalanı ifade etmektedir. O halde  ≡  (mod ) ise,  ∈ Z olmak üzere,
 =  +  olur.  ≡  (mod ) ifadesine denklik ya da kongruans adı verilir.
Örneğin, 100! ≡? (mod 11) matematiksel ifadesini, "100! sayısının 11’e bölümün­
den kalan kaçtır?" şeklinde ifade edebiliriz. Benzer şekilde, 222
· · · 2} sayısının 9’a
| {z
100 tane
bölümünden kalanın bulunmasını, 222
· · · 2} ≡? (mod 9) ile ifade edebiliriz. Buna
| {z
100 tane
göre, 9’a bölünme kuralı göz önüne alınırsa,
222
· · · 2} ≡ 2 · 100 = 200 ≡ 2 (mod 9)
| {z
100 tane
yazılabilir.
Not : Bir sayının son rakamını bulmak için mod 10’a, son iki rakamını bulmak için
mod 100’e, son üç rakamını bulmak için mod 1000’e vb. bakılır.
Örnek 51 41000 sayısının son rakamı kaçtır?
Çözüm : 41000 ≡? (mod 10) denkliğini hesaplamalıyız. Bunun için,
41 ≡ 4 (mod 10)  42 ≡ 6 (mod 10)  43 ≡ 4 (mod 10)  44 ≡ 6 (mod 10)
olacağından, 4’ün çift kuvvetlerinde son rakam 6, tek kuvvetlerinde ise son rakam
4’tür. O halde, 41000 sayısının son rakamı 6 olur.
Örnek 52 20112012 sayısının 7’ye bölümünden kalan nedir?
Çözüm : 2011 = 7 · 287 + 2 olduğundan, dolayı
20112012 ≡ 22012 (mod 7)
yazabiliriz. Şimdi, 2’nin kuvvetlerini mod 7’de inceleyerek, 1 veya periyodik olun­
caya devam edelim.
21 ≡ 2 (mod 7)  22 ≡ 4 (mod 7)  23 ≡ 1 (mod 7)
olduğundan, 2’nin 3’ün katı olan tüm kuvvetleri mod 7’de 1 kalanını verecektir. Buna
göre,
¡ ¢670·3 2
· 2 ≡ 1 · 4 ≡ 4 (mod 7)
22012 = 22010 · 22 = 23
elde edilir. Yani, kalan 4 olarak bulunur.
35
Temel Bilgiler
Örnek 53 13 +23 +33 +43 + · · · + 20133 sayısının 8’e bölümünden kalan
kaçtır?
Çözüm :  tek ise, 3 ≡  (mod 8) ve  çift ise, 3 ≡ 0 (mod 8) olduğundan,
istenen kalan
13 + 33 + 53 + · · · + 20133
sayısının 8’e bölümünden kalana eşittir. Kalanlar 1 3 5 7 şeklinde periyodik olarak
devam edeceğinden, ve her periyottaki sayıların toplamı 8’e tam bölüneceğinden,
sadece en son sayılardan dörtlü oluşturmayan sayıları incelemek yeterlidir.
20133 ≡ 5 (mod 8) olduğundan,
13 + 23 + 33 + 43 + · · · + 20133
elde edilir.
≡ 20093 + 20113 + 20133
≡ 1+3+5
≡ 1 (mod 8)
Örnek 54 2 +27 sayısı 7’ye bölünecek şekilde 100’den küçük kaç pozitif n tam­
sayısı vardır?
Çözüm :
21 ≡ 2 (mod 7) 
22 ≡ 4 (mod 7) 
23 ≡ 1 (mod 7)
olduğundan  = 3 olduğunda,
2 + 27 ≡ 1 + 27 ≡ 0 (mod 7)
olur. O halde 100’den küçük 3 formunda olan sayıların sayısını bulmalıyız. Bunlar,
3 6 9  99
sayıları olduğundan, (99 − 3) 3 + 1 = 33 tanedir.
Örnek 55 İlk 98 sayının 1234...9798 biçiminde art arda yazılmasıyla elde edilen
sayının 45’e bölümünden kalan kaçtır?
Çözüm : Verilen sayının rakamları toplamı
98 · 99
= 99 · 49
2
olduğundan, sayı 9’a tam bölünür. Sayının son rakamı 8 olduğundan, 5’e bölündüğünde,
3 kalanını verir. Buna göre, kalanın 9’un katı ve 5’e bölündüğünde 3 kalanını veren
bir sayı olması gerekir. Bu sayı 18’dir.
1 + 2 + 3 + · · · + 98 =
36
Matematik Olimpiyatlarına Hazırlık 1
Örnek 56 1 ile 701 arasında alınan pozitif tamsayılarla oluşturulan (x, y) sıralı
ikililerinden kaçı için x2 +y 2 sayısı 49’a kalansız bölünür? (Doğuş Ün. Mat.Y. ­
2007)
Çözüm : 2 +  2 ifadesi 49’a tam bölünüyorsa 7’ye de tam bölünür. Bir sayının
karesini mod 7’de inceleyelim.
02 ≡ 0 (mod 7)  12 ≡ 1 (mod 7)  22 ≡ 4 (mod 7)  32 ≡ 2 (mod 7) 
42 ≡ 2 (mod 7)  52 ≡ 4 (mod 7)  62 ≡ 1 (mod 7)
elde edilir. Buna göre, 2 +  2 ifadesinin mod 7’de 0 olabilmesi için, yani 7’ye tam
bölünebilmesi için, hem 2 hem de  2 ifadelerinin 7’ye tam bölünebilmesi gerekir.
Bunun için de hem  hem de  sayıları 7’nin katı olmalıdır. Bu durumda, 2 +  2
sayısının 49’a bölüneceği açıktır. Böylece, 1 ile 701 arasında 7 ile bölünen 100 tane
 sayısı ve 100 tane de  sayısı bulunacağından, istenen şekilde 100 · 100 = 104 tane
( ) ikilisi olduğu görülür.
Örnek 57
√
4k + 3 sayısı tamsayı olacak şekilde kaç k pozitif tamsayısı vardır?
Çözüm : Verilen ifadenin tamsayı olması için, 4 + 3 sayısının bir tamkare olması
gerekir. Bu tamkarenin de 4’e bölündüğünde 3 kalanını vermesi gerekir. O halde, şu
soruya cevap arayalım. "Bir sayının karesi mod 4’de kaç olabilir?".
02 ≡ 0 (mod 4),
12 ≡ 1 (mod 4),
22 ≡ 0 (mod 4) ,
32 ≡ 1 (mod 4) 

olduğundan, bir sayının
karesi 4’e bölündüğünde kalan 0 veya 1 olabilir. Yani, 3
√
olamaz. O halde, 4 + 3 ifadesi asla bir tamkare olamaz.
Örnek 58 7 ≡ 1 (mod 101) denkliğini sağlayan bir x sayısı bulunuz.
Çözüm : Bu soruyu biraz önceki örneklerde olduğu gibi tek tek hesaplayarak yapmak
zordur. Bunu hesaplamış olsaydık,
71 ≡ 7 (mod 101)  72 ≡ 49 (mod 101)  73 ≡ 40 (mod 101)  
şeklinde devam ederek, 7100 ≡ 1 (mod 101) olduğunu görebilirdik.
Açıktır ki, bu bizim oldukça çok vaktimizi alır. Bunun yerine, Fermat Teoremi de
denilen, bir  asal sayısı için,
−1 ≡ 1 (mod )
denkliğinden de yararlanmak daha kolay olabilir. Bu durumda, 7100 ≡ 1 (mod 101)
olduğunu doğrudan söyleyebilirdik.
37
Temel Bilgiler
I Fermat Teoremi :  bir asal sayı ise,
 ≡  (mod )
’dir. Eğer, EBOB( ) = 1 ise, −1 ≡ 1 (mod ) olur.
Örnek 59 13200 sayısının 19’a bölümünden kalan kaçtır?
Çözüm : Fermat teoreminden, 1318 ≡ 1 (mod 19) diyebiliriz. Buna göre,
¡
¢11
2
13200 = 1318
· 132 ≡ 132 ≡ (−6) = 36 ≡ 17 (mod 19)
elde edilir.
Örnek 60 10100 sayısının 65’e bölümünden kalan kaçtır?
Çözüm : 65 = 13·5 olduğundan, 10100 sayısının önce 13’e, sonra da 5’e bölümünden
kalanı bulalım. 10100 ≡? (mod 13) denkliğini Fermat Teoremini kullanarak bulabili­
riz. 1012 ≡ 1 (mod 13) olduğundan,
¡
¢8
10100 = 1012·8+4 = 1012 104 ≡ 104 = 1002 ≡ 42 ≡ 3 (mod 13)
elde edilir. Diğer yandan, 10100 sayısının 5’e bölümünden kalan 0’dır. O halde, 10100
sayısının 65’e bölümünden kalan sayı, 13’e bölündüğünde 3 kalanını veren ve 5’in
katı olan bir sayıdır. 13 + 3 formundaki sayılardan, 5’in katı olan 65’ten küçük tek
sayı 55’tir. O halde, 10100 sayısının 65’e bölümünden kalan 55 bulunur.
F Aşağıdakileri Fermat Teoremini Uygulayarak Hesaplayınız.
1. 516 ≡? (mod 17) (Yanıt : 1)
2. 320 ≡? (mod 17) (Yanıt : 13)
3. 781 ≡? (mod 17) (Yanıt : 7)
4. 10100 sayısının 33’e bölümünden kalan kaçtır? (Yanıt : 1)
5. 11111 sayısının 65’e bölümünden kalan kaçtır? (Yanıt : 31)
I Fermat Teoremi bize her zaman denkliği sağlayan en küçük üssü vermeyebilir.
Daha küçük bir üs varsa Fermat Teoremi ile elde edilen üssü bölmesi gerekir.
Örnek 61 5 ≡ 1 (mod 13) denkliğini sağlayan en küçük n sayısı kaçtır?
Çözüm : Fermat teoremine göre, 512 ≡ 1 (mod 13) olduğunu hemen söyleyebiliriz.
5’in daha küçük bir üssünde bu denklik sağlanıyorsa, bu üssün 12’yi bölmesi gerekir.
Gerçekten, 54 ≡ 1 (mod 13)’tür.
I Burada aklımıza şu soru gelecektir. Fermat teoremini sadece mod, yani  sayısı
asal iken kullanabiliriz. Eğer, mod asal olmazsa ne yapacağız? Bunun da çözümü
var. Bu durumda da Euler Teoremi dediğimiz teoremi kullanacağız. Bu teorem­
lerle ilgili detaylı bilgileri ve değişik soruları Matematik Olimpiyatlarına Hazırlık 3
kitabında bulabilirsiniz. Burada sadece bu teoremlere kısaca değinilmiştir.
38
Matematik Olimpiyatlarına Hazırlık 1
1.7
Basit Denklem Çözümleri ve Sayıların Özelliklerinin
Kullanılması
Sayının tamsayı veya pozitif tamsayı olmasının, tek veya çift olmasının, asal
olmasının, ardışık sayılar olmasının aşağıdaki denklemlerde nasıl kullanıldığını ince­
leyiniz. Bu örneklerdeki yorumların, sadece verilen sayı kümeleri ve bu kümelerin
altkümeleri için geçerli olduğunu ve başka bir sayı kümesinde benzer yorumların
yapılamayacağını görünüz.
Örnek 62 n (n + 1) = p denklemini sağlayan tüm p asal sayılarını ve n ve k
pozitif tamsayılarını bulunuz.
Çözüm :  ile  + 1 sayıları ardışıktır ve biri tek biri çifttir. O halde, bu denklemin
sağlanması için, eşitliğin sağ tarafındaki sayının çift olması gerekir ki,  asal olduğun­
dan sadece 2 olabilir. Bu durumda,  ( + 1) = 2 olur. Bu eşitliğin sağlanabilmesi
sadece  = 1 durumunda mümkündür. Çünkü,   1 durumunda, sol tarafın bir tek
çarpanı vardır, oysa sağ taraf 2’nin bir kuvvetidir. Buna göre,  = 1 yazılırsa, 2 = 2
olur. Buradan  = 1 elde edilir.
Örnek 63 x, y ve z asal sayılar olmak üzere,
½
x + y + z = 20
y + 2x = z
denklem sisteminin kaç çözümü vardır?
Çözüm : Üç asal sayının toplamının bir çift sayı olması için sayılardan biri mutlaka
2 olmalıdır. Aksi durumda, üçü de tek asal sayı olursa toplam da tek sayı olacaktır.
Diğer taraftan,  bir çift sayı olamaz, çünkü,  çift olursa sistemin ikinci denklemine
göre  de çift olmak zorunda olur. O halde  = 2 olmalıdır. Bu değer denklemlerde
yerine yazılırsa,  +  = 18 ve  + 4 =  denklemleri elde edilir. Bu denklemlerin
çözümünden,  = 11 ve  = 7 bulunur.
Örnek 64 n, m ∈ Z+ olmak üzere 5n + 3m = 100 denklemini çözünüz.
Çözüm : Denklemde, 5 ve 100 terimleri 5’in katıdır. O halde, 3 terimi de 5’in
katı olmalıdır. Bu durumda, , 5’in katı olmalıdır. Denklemi daha basit hale getirmek
için,  ∈ Z+ için,  = 5 yazalım. Böylece, denklem daha basit olarak,
20 − 
 + 3 = 20 veya  =
3
şekline dönüşür.  ve  sayıları pozitif tamsayı olduğundan,  = 2 5 8 11 14 veya
17 olabilir.
Not : Burada, pozitif tamsayılar kümesi yerine tamsayılar kümesi alınsaydı, sonsuz
sayıda  sayısı elde ederdik. Yani, denklemin verildiği sayı kümesi problemdeki en
önemli noktalardan biridir.
39
Temel Bilgiler


= eşitliğinde,  ile  aralarında asal


ve  ile  de aralarında asal ise,  =  ve  =  olmak zorundadır.
F    ve  pozitif tamsayıları için,
2
Örnek 65 n ∈ Z+ ve p, q asal sayılar ise, p (2n) = q 6 (2n − 1) denklemini
çözünüz.
Çözüm : Verilen denklemi
(2)2
6
=
2 − 1

şeklinde yazalım. Sol taraftaki, 2 ve 2 − 1 sayıları ardışık olduklarından aralarında
asaldır. Sağ tarafta da  ve  asal olduğundan, sadeleşemezler. O halde,
 6 = (2)2 ve  = (2 − 1)
olmalıdır.  6 = (2)2 eşitliğinde, sağ taraf çift ve  asal olduğundan,  sayısı bir çift
asal sayı yani 2 olmalıdır. Bu durumda, 26 = 42 , eşitliğinden,  = 4 olur.  = 4
ise,  = 7 olur.
Örnek 66 a, b ve c ∈ Z+ için, a + 9b + 20c = 50 denklemini sağlayan kaç
(a, b, c) üçlüsü vardır?
Çözüm :   ve  sayıları pozitif tamsayı olduklarından,  sayısı 3’ten küçük ol­
malıdır. Yani,  = 1 veya  = 2 olabilir.
Eğer,  = 1 ise,  + 9 = 30 olur. Bu denklemde de,  sayısı 1, 2 veya 3 olabilir.
Eğer,  = 1 ise,  = 21 ,  = 2 ise  = 12 ve  = 3 ise  = 3 olur.
Eğer,  = 2 ise,  + 9 = 10 olur. Bu denklem ise sadece  = 1 ve  = 1
durumunda sağlanır.
Böylece, (  ) üçlüsü, (21 1 1)  (12 2 1)  (3 3 1) ve (1 1 2) olabilir. Yani,
4 tanedir. Yine, bu soruda da, pozitif tamsayılar yerine tamsayılar kümesi alınsaydı,
sorunun sonsuz sayıda çözümü olurdu.
1
1
1
− = denkleminin pozitif tamsayılarda kaç çözümü vardır?
x
y
3
Çözüm : Denklem düzenlenirse 3 − 3 =  eşitliğinden,
3 + 9 − 9
9
3
=
=3−
=
+3
+3
+3
Örnek 67
elde edilir. Buradan  + 3 = 9 ve  = 6 olabilir ki bu durumda  = 2 elde edilir.
Yani denklemin sadece 1 çözümü vardır ve bu çözüm (2 6)’dır.
40
Matematik Olimpiyatlarına Hazırlık 1
Örnek 68 7 (x + y) = xy denklemini sağlayan kaç (x, y) tamsayı ikilisi vardır?
Çözüm : 7 ( + ) =  eşitliğinden, ( − 7) ( − 7) = 49 olur. 49 sayısının
tamsayı bölenleri
−49 −7 −1 1 7 ve 49
olduğundan ( ) tamsayı çiftinin bir çözüm olabilmesi için ( )’nin
(0 0)  (14 14)  (−42 6)  (6 −42)  (8 56)  (56 8)
ikililerinden biri olması gerekir. O halde 6 tane ( ) tamsayı ikilisi vardır.
Örnek 69 a ve b pozitif tamsayıları için, (a + 2b) (a + 4b) = 24 olacak şekilde
kaç (a, b) ikilisi vardır?
Çözüm : ( + 2) ( + 4) ifadesinin çarpanlarının her ikisi de çift veya her ikisi de
tektir. Buna göre,
24 = 4 · 6 = 3 · 8 = 2 · 12 = 1 · 24
olduğundan,  + 4   + 2 olduğu da göz önüne alınırsa,
½
½
 + 4 = 6
 + 4 = 12
veya
 + 2 = 4
 + 2 = 2
denklem sistemleri elde edilir. Fakat, ikinci denklem sistemini de doğrudan eleyebili­
riz. Çünkü,  ve  pozitif tamsayılar olduğundan, +2 = 2 olması mümkün değildir.
Buna göre, ilk denklem sisteminden,  = 1 ve  = 2 elde edilir. O halde bu denklemi
sağlayan tek pozitif tamsayı ikilisi ( ) = (2 1) olur.
F 2 − 2 = ( − ) ( + ),
¡
¢
F 3 − 3 = ( − ) 2 +  + 2 ,
¡
¢
F 3 + 3 = ( + ) 2 −  + 2 .
Örnek 70 n ve n + 100 sayılarının her ikisinin de pozitif bölenlerinin sayısı tek
olacak şekilde kaç değişik n pozitif tamsayısı vardır?
Çözüm : Bir sayının pozitif bölenlerinin sayısının tek olması için sayı tamkare ol­
malıdır. O halde
 = 2 ve  + 100 = 2 + 100 =  2
olmalıdır. Buradan
 2 − 2 = ( − ) ( + ) = 100 = 22 52
eşitliğine göre her iki çarpanın da çift olması gerektiği göz önüne alınırsa
 +  = 50 ve  −  = 2
eşitliklerinden  = 26 ve  = 24 elde edilir. O halde  = 242 = 576 elde edilir.
Yani istenen şekilde bir sayı vardır.
41
Temel Bilgiler
Örnek 71 Kendisiyle, kendisinin 2 fazlasının toplamı 2’nin bir kuvvetine eşit olan
100’den küçük kaç pozitif tamsayı vardır?
Çözüm : Sayının kendisi  ise, 2 fazlası  + 2 olur. Buna göre,
 + ( + 2) = 2 yani, 2 ( + 1) = 2
eşitliği sağlanmalı. Bu eşitliğe göre,  + 1 sayısı da 2’nin bir kuvveti olmalıdır. Bu
ise,  = 1 3 7 15 31 63 sayıları için doğrudur.
Örnek 72 a2 +b = b1999 denklemini sağlayan kaç (a, b) tamsayı ikilisi vardır?
(Estonya M.O. 1999)
¡
¢
Çözüm : 2 =  1998 − 1 yazalım.  ≥ 2 için
 ve 1998 − 1
sayıları aralarında asal olacağından eşitliğin sol tarafının tamkare olması nedeniyle,
bu sayıların her ikisi de tamkare olmalıdır. Fakat 1998 sayısı zaten bir tamkare
olduğundan 1998 − 1 sayısı tamkare olamaz. Dolayısıyla  ≥ 2 olamaz. Yani,
 ≤ 1 olmalıdır. Buna göre,
 = 1 ise,  = 0
 = 0 ise,  = 0
 = −1 ise,  = 0
olur. Ayrıca,  ≤ −2 olamaz. Çünkü bu durumda da, 2  0 olur. O halde denklemin
sağlayan ( ) pozitif tamsayı ikilileri,
olmak üzere, 3 tanedir.
(0 1)  (0 0) ve (0 −1)
Örnek 73 1 + 2 + 3 + · · · + n ifadesi tamkare olacak şekilde 100’den küçük
en büyük n pozitif tamsayısı kaçtır?
 ( + 1)
Çözüm :  ∈ Z olmak üzere, 1+2+3+· · ·+ =
= 2 eşitliğini sağlayan
2
100’den küçük en büyük  pozitif tamsayısını arıyoruz.  sayısı ile  + 1 sayıları
aralarında asal olduğundan çarpımlarının tamkare olması için biri tamkare olmalı,
diğeri ise 2’ye bölündüğünde tamkare olmalıdır. Buna göre,  ve ( + 1) 2 tamkare
olacak şekildeki sayıları arayalım.  + 1 çift olması gerektiğinden  tek tamkareler
olmalıdır. 92 için ( + 1) 2 tamkare değildir.  = 72 için, ( + 1) 2 = 25 olur ki,
istenen koşulu sağlar. Bu durumda,  = 35 olur. Şimdi de,  + 1 ve 2 tamkare
olacak şekildeki sayıları inceleyelim. 2 sayısının bir tamkare olması için,  sayısı
2’nin bir tek kuvveti olmalıdır. Bunu sağlayan 100’den küçük tek sayı  = 8’dir. Biz,
istenen koşulu sağlayan 100’den küçük en büyük  tamsayısını aradığımızdan yanıt
 = 49 bulunur.
42
Matematik Olimpiyatlarına Hazırlık 1
Örnek 74 x ve y rasyonel sayılarının en sadeleşmiş durumlarında paydaları sıra­
sıyla 60 ve 70’tir. Buna göre, x + y sayısı, sadeşleşmiş durumda iken paydası en
küçük kaç olabilir? (Harvard Math. Tourn. 2003)
Çözüm :  ve  rasyonel sayılarının en sade halleri, sırasıyla 60 ve 70 olsun.
Buna göre,


7 + 6
+ =
+
=
60 70
420
ifadesinde, paydanın en küçük olması için, 7 + 6 ile 420’yi sadeleştirebilecek en
büyük sayıyı bulmalıyız.  sayısının 60 ile,  sayısının da 70 ile aralarında asal olduğu
unutulmamalıdır. Buna göre, 7 + 6 sayısı kesinlikle 2, 3 ve 7 ile bölünemez. Bunu
biraz daha açıklayalım.
7 + 6 sayısında, 6 sayısı hem 2 hem de 3 ile bölünür. Fakat, 7 sayısı ne 2 ne de
3 ile bölünemez. O halde, 7 + 6 sayısı asla 2 ve 3 ile sadeleşemez. Benzer şekilde,
7 + 6 sayısında, 7 sayısı 7 ile bölünürken, 6 sayısı asla 7 ile bölümez. Çünkü, 
ile 70 aralarında asal idi. Böylece, her ne olursa olsun, 7 + 6 sayısı, 2 3 ve 7 ile
sadeleşmeyecektir. Böylece, 420 = 22 ·3·5·7 olduğundan dolayı, 7+6 sayısı sadece
5 ile sadeleşebilecektir.  = 1 ve  = 3 alınması durumunda, 7 + 6 = 7 + 18 = 25
olacaktır ve
3
5
1
+
=
60 70
84
eşitliğinden, yanıt 84 olarak bulunur.
Örnek 75 n2 +4n − 7 ifadesi tamkare olacak şekilde kaç n tamsayısı vardır?
Çözüm :  ∈ Z için, 2 + 4 − 7 = 2 olsun. Bu ifadeyi
biçiminde yazalım. Buradan,
olacaktır. İki kare farkından,
2 + 4 + 4 − 11 = 2
( + 2)2 − 2 = 11
( + 2 − ) ( + 2 + ) = 11
elde edilir. Bu eşitliğin sağlanması için,
½
½
 −  + 2 = 1
 −  + 2 = 11
,
 +  + 2 = 11
++2=1
½
½
 −  + 2 = −1
 −  + 2 = −11
,
,
 +  + 2 = −11
 +  + 2 = −1
olmalıdır. Bu denklem sistemlerinden de,  = 4 veya  = −8 elde edilir.
43
Temel Bilgiler
Örnek 76 x, y, z, w ∈ R olmak üzere,
⎧
2x + y + z + w
⎪
⎪
⎨
x + 3y + z + w
x + y + 4z + w
⎪
⎪
⎩
x + y + z + 5w
=
=
=
=
1
2
3
25
denklem sistemi veriliyor. w değeri kaçtır? (Harvard MIT Math. Tournament 2002)
Çözüm : Denklem sistemindeki denklemleri sırasıyla, 12 6 4 ve 3 ile çarpalım.
Böylece,
⎧
24 + 12 + 12 + 12 = 12
⎪
⎪
⎨
6 + 18 + 6 + 6 = 12
4 + 4 + 16 + 4 = 12
⎪
⎪
⎩
3 + 3 + 3 + 15 = 75
elde edilir. Denklemleri taraf tarafa toplarsak,
37 + 37 + 37 + 37 = 111
elde edilir. Buradan da,  +  +  +  = 3 bulunur. Şimdi, dördüncü denklemde bunu
kullanırsak, 3 + 4 = 25 eşitliğinden,  = 112 bulunur.
¡
¢
Örnek 77 q p2 +p = 15r − 3pr eşitliğini sağlayan kaç (p, q, r) asal sayılar
üçlüsü vardır?
Çözüm : Verilen eşitliği,
 ( +  + 3) = 3 · 5 · 
şeklinde yazabiliriz.
 = 3 ise, (3 +  + 3) = 5 ·  ⇒ 4 = 2 olmalıdır. Bu eşitliği sağlayan  ve 
asal sayıları olamaz. Çünkü, her ikisi de çift olmalı ve birbirine eşit olmamalı.
 = 5 ise, (5 +  + 3) = 3 ·  ⇒ 6 = 0 olduğundan,  = 0 olur ki, buradan da
çözüm elde edilemez.
 =  ise, ( +  + 3) = 3 · 5 eşitliğinden,
15 − 3
−3 − 3 + 18
18
=
=
= −3 +
+1
+1
+1
olması gerekir. Bu eşitliğe göre, +1 sayısı 18’in böleni olmalıdır.  asal olduğundan,
2 veya 17 olabilir.
 = 2 ⇒  = 3 ve  = 17 ⇒  = −2
olur. O halde, tek çözüm (  ) = (2 3 2) bulunur.
44
Matematik Olimpiyatlarına Hazırlık 1
1.8
Basit Eşitsizlikler
Eşitsizliklerle ilgili temel bilgilerimizi hatırlayalım.
I   0 olması durumunda,  tek ise,   0 ve  çift ise,   0 olur.
I Her  ∈ R için, 2 ≥ 0’dır.
⎧ 2
⎨  ≤ 1 ise,  ∈ [−1 1] olmalıdır.
2 ≥ 2 ise,  ≥  veya  ≤ − olmalıdır.
⎩ 2
 ≤  ise,  ∈ [0 1] olmalıdır.
I    olsun. Bu durumda,
⎧


⎪
⎨   0 ise    ve



⎪
⎩   0 ise    ve   


olur.
1
1
I 0     ise
 ve    ( ∈ Z+ ) olur.


1
1
I     0 ise
 ve  ∈ Z+ için,  tek ise,    ve  çift ise,   


olur.
I 2 + 2 + 2 = 0 eşitlğinin sağlanması için  =  =  = 0 olmalıdır.
Örnek 78 x2 −y < −3 ve x2 +y < 7 eşitsizliklerini sağlayan kaç (x, y) tam­
sayı ikilisi vardır?
Çözüm : Eşitsizlikleri taraf tarafa toplarsak, 22  4 olur. Buna göre 0 ≤ 2  2
yani,  ∈ {0 1 −1} olur. Diğer taraftan, 2 + 3    7 − 2 olduğundan,  = 0
olduğunda,  ∈ {4 5 6} ve  = ±1 olduğunda da  = 5 olacağından, 5 tane ( )
ikilisi vardır.
1
Örnek 79 a2 ≤ a ve b2 ≤ 1, 2 ≤ 1 olduğuna göre, a + b = c olacak şekilde
c
kaç  tamsayısı vardır?
Çözüm : İlk eşitsizlikten 0 ≤  ≤ 1 2 ≤ 1 eşitsizliğinden −1 ≤  ≤ 1 ve
1
≤ 1 eşitsizliğinden de 2 ≥ 1, yani  ≥ 1 veya  ≤ −1 eşitsizlikleri elde edilir.
2
0 ≤  ≤ 1 ve −1 ≤  ≤ 1
eşitsizliklerinden, −1 ≤  +  ≤ 2 olur. Buna göre,  +  ∈ {−1 0 1 2} olur.  ≥ 1
veya  ≤ −1 olduğundan,  +  =  eşitliğini sağlayan  tamsayıları, −1 1 ve 2
olabilir.
45
Temel Bilgiler
Örnek 80 x, y pozitif tamsayılar olmak üzere,
x
1
10
5
2 < < 3 ve <
<
y
2
y
2
koşullarını sağlayan (x, y) ikililerinin sayısı kaçtır?
Çözüm : İkinci eşitsizliği düzenlersek,

2

10
5
olur. Buradan da, 4    20 elde edilir. Yani,  ∈ {5 6  19} olur. Diğer taraftan,
2    3 eşitsizliğinde,
 = 5 için, 10    15 ise,  ∈ {11 12 13 14};
 = 6 için, 12    18 ise,  ∈ {13 14 15 16 17};
 = 7 için, 14    21 ise,  ∈ {15 16 17 18 19 20};
···
 = 19 için, 38    57 ise,  ∈ {39 40  55 56} olacaktır. Yani,
 sırasıyla, 4 5  18 farklı tamsayı değerleri alabilir. Buna göre, istenen koşulu
sağlayan
18 + 4
4 + 5 + 6 + · · · + 18 =
(18 − 4 + 1) = 165
2
( ) tamsayı ikilisi vardır.
2
n
7
8
<
<
eşitsizliğini, sadece bir k tamsayısı için sağlayan
15
n+k
13
en büyük n pozitif tamsayısı kaçtır? (AIME 1987)
8

7
Çözüm :


eşitsizliğinin sol ve sağ tarafından,
15
+
13
8 + 8  15 ve 13  7 + 7
Örnek 81
olur. Bu iki eşitsizlikten de,
7
6

7
8
µ
¶
6 7
7
6

aralığında
−
=
olduğundan,   112 olduğunda,

8
7
56
7 8
kesinlikle iki tamsayı olacaktır.  = 112 alınırsa,
7 · 112
6 · 112

7
8
eşitsizliğinden 96    98 elde edilir. Bu aralıkta, sadece bir  tamsayısı vardır. O
halde, en büyük  pozitif tamsayısı 112’dir.
bulunur.
46
Matematik Olimpiyatlarına Hazırlık 1
1.9
Faktöriyel Kavramı
1’den ’ye kadar olan sayıların çarpımını ! ile gösteririz. 0! = 1 olarak tanım­
lanır. Örneğin,  ≥ 3 için,
! =  · ( − 1) · ( − 2) · ( − 3)!
şeklinde yazılabilir. Aşağıda, faktöriyel içeren denklemlerle bazı ilginç soruları ince­
leyiniz.
Örnek 82 k ve n pozitif tamsayılar olmak üzere, ((3!) !) ! = 3! · k · n! ise, k + n
sayısının alabileceği en küçük değer kaçtır? (AIME 2003)
Çözüm : ((3!)!)! = (6!)! = 720! = 720 · 719! = 3! · 120 · 719! olduğundan,  = 120
ve  = 719’dur. O halde,  +  sayısının alabileceği en küçük değer 120 + 719 = 839
olur.
21!22!23! · · · 30!
ifadesinin
11!12!13! · · · 20!
a) bir rasyonel sayının karesi olması için,
b) bir tamkare olması için,
en küçük hangi pozitif tamsayıyla çarpılması gerekir?
Örnek 83
2
2
2
2
2
21!22! · · · 30!
(21!) 22 · (23!) 24 · (25!) 26 · (27!) 28 · (29!) 30
=
11!12! · · · 20!
(11!)2 12 · (13!)2 14 · (15!)2 16 · (17!)2 18 · (19!)2 20
şeklinde yazabiliriz. Paydadaki kareli ifadeler kesrin payındaki kareli ifadelerin içinde
olduğundan dolayı sadeleştiğinde yine tamkare olacaktır. O halde, bu ifadede ki tüm
tamkareleri  2 ile gösterirsek,
Çözüm :
= 2
 2 · 11 · 13
22 · 24 · 26 · 28 · 30
=
12 · 14 · 16 · 18 · 20
2·2·3
olur ki, olur ki,  2 bir çift sayı olduğundan dolayı, bu sayının hem bir rasyonel sayının
karesi, hem de bir tamsayının karesi olması için 11 · 13 · 3 ile çarpılması yeterlidir.
Örnek 84 1! + 2! + 3! + · · · + 100! sayısının son rakamı kaçtır?
Çözüm : 5!’den sonraki tüm faktöriyellerin son rakamı 0 olduğundan, sadece,
1! + 2! + 3! + 4!
toplamının son rakamını hesaplamak yeterlidir. 1 + 2 + 6 + 24 = 33 olduğundan, son
rakam 3 bulunur.
47
Temel Bilgiler
Örnek 85 5 · 10 · 15 · 20 · · · 500 sayısının
a) sağdan 97’inci rakamı kaçtır?
b) sağdan 98’inci rakamı kaçtır?
Çözüm : a) Bu sayının sonunda kaç sıfır olduğunu hesaplayalım.
5 · (2 · 5) · (3 · 5) · (4 · 5) · · · (100 · 5) = 5100 · (1 · 2 · 3 · · · 100)
= 5100 · 100!
eşitliğine göre, yeterince 5 olduğundan 100! içinde kaç tane 2 çarpanı olduğunu bu­
lalım.
100 : 2 = 50 50 : 2 = 25 25 : 2 = 12 12 : 2 = 6 6 : 2 = 3 3 : 2 = 1
olduğundan,
50 + 25 + 12 + 6 + 3 + 1 = 97
tane 2 çarpanı vardır. O halde, 5100 · 100! sayısının sonunda 97 tane sıfır vardır. Buna
göre, 97’inci rakam 0 olur.
b) Sayının tüm sıfırları atıılırsa, geriye 0 haricindeki sayılar ve 53 ’ün çarpımıyla elde
edilen bir sayı kalmıştır. 5 sayısının 0 haricindeki bir sayıyla çarpımı daima 5’in katı
olacağından 98’inci rakamı 5 olacaktır.
Örnek 86 xy iki basamaklı bir sayı olduğuna göre, x! + y! = xy denkleminin
kaç çözümü vardır?
Çözüm : 5! = 120 olduğundan,  ve  rakamları 5’ten küçük olmalıdır. Bu özelliğe
sahip tüm sayılardan, yani 10 11 12  44 sayılarından hiçbirinin verilen denklemi
sağlamadığı görülebilir. Yani, denklemin çözümü yoktur.
Örnek 87 x, y ve z karışık olarak verilmiş ardışık negatif olmayan tamsayılar
olmak üzere,
x! = y! + z!
eşitliğini sağlayan kaç (x, y, z) üçlüsü vardır?
Çözüm : ’in bu ardışık sayıların en büyüğü olacağı açıktır.  =  + 1 denilirse, 
ve , sırasıyla,  − 1 ve  olacaktır. Buna göre, sırasıyla,
( + 1)! = ! + ( − 1)!
( + 1)  ( − 1)! =  ( − 1)! + ( − 1)!
2 +  =  + 1
olur. En son elde edilen 2 = 1 eşitliğinden  = 1 elde edilir. O halde, (  ) 
(2 1 0) veya (2 0 1) olabilir.
48
Matematik Olimpiyatlarına Hazırlık 1
Örnek 88 n! = a! + b! + c! denkleminin kaç çözümü vardır? (KANADA M.O
1983)
Çözüm : Denkleme göre,  ,    olacağı açıktır. Buna göre,    ≤  − 1 ve
dolayısıyla da,
! + ! + ! ≤ 3 ( − 1)!
olacaktır.   3 için, !  3 ( − 1)! olacağından,   3 için çözüm yoktur (
sayısının nasıl sınırlandığını inceleyiniz.)
O halde,  = 1 2 veya 3 olabilir. Kontrol edilirse, 3! = 2! + 2! + 2! olduğundan
 = 3 ’ün denklemin bir çözümü olduğu görülür.
Örnek 89 Her bir terimi 15’ten küçük veya eşit olan a1 , a2 , ..., a pozitif tam­
sayı dizisi için, a1 ! + a2 ! + · · · + a ! ifadesinin son dört rakamının 2001 olması
için, n en küçük kaç olmalıdır? (CENTRO Amerikan M.O 2001)
Çözüm : 15’ten küçük sayıların faktöriyellerinin son dört rakama kadar değerlerini
yazalım.
1! 7→ 1 2! 7→ 2 3! 7→ 6 4! 7→ 24 5! 7→ 120, 6! 7→ 720,
7! 7→ 5040, 8! 7→ 0 320 9! 7→ 2880, 10! 7→ 8800, 11! 7→ 6800,
12! 7→ 1600, 13! 7→ 0 800, 14! 7→ 1200, 15! 7→ 8000
Faktöriyellerin son dört rakamının toplamının 2001 olmasını istiyoruz. Görüldüğü
gibi tüm faktöriyellerde son rakamı tek olan sayı sadece 1 olduğundan 1 kesinlikle
olmalıdır. Geriye 2000 toplamı kalır. Bulduğumuz eşitliklerden görülür ki, son dört
rakamın toplamı 2000 olabilecek sayılar sadece 13! ve 14! olabilir. O halde,  en
küçük 3 olur.
Not : Bu soruda, terimlerin 15’ten küçük olması şartı kaldırılsaydı, cevap 2 olacaktı.
Çünkü, 19! sayısının son dört rakamı 2000’dir ve 19! + 1! sayısının son dört rakamı
da 2001 olur.
F Ardışık  tane tamsayının çarpımı 1’den ’ye kadar ( dahil) tüm sayılar ile tam
bölünür. Hatta ! ile bölünür.
Örnek 90 n pozitif tamsayı olmak üzere n5 −5n3 +4n sayısının daima 120’ye
bölüneceğini gösteriniz.
Çözüm : Verilen ifadeyi çarpanlarına ayıralım.
¡
¢
¡
¢¡
¢
5 − 53 + 4 =  4 − 52 + 4 =  2 − 1 2 − 4
= ( − 2) ( − 1)  ( + 1) ( + 2)
olur. Yani 5 − 53 + 4 sayısı ardışık 5 sayının çarpımıdır ve 5! = 120’ye bölünür.
49
Temel Bilgiler
Örnek 91 n ∈ Z+ olmak üzere, (n + 10) (n + 11) · · · (n + 44) sayısının 315 ’e
bölünebildiğini ama, 316 ’ya bölünemeyebileceğini gösteriniz.
Çözüm : 44 − 10 + 1 = 35 olduğundan bu çarpım daima 35!’e tam bölünecektir. 35!
içinde 1 tane 27’ye, 3 tane 9’a, ve 11 tane 3’e bölünen çarpan vardır. Yani verilen
ifade kesinlikle 31+3+11 = 315 ’e bölünür. Diğer yandan,
( + 44)!
( + 10) ( + 11) · · · ( + 44) =
( + 9)!
ifadesinde, paydadaki ifadeyi 27 olacak şekilde  = 18 alırsak, 62!27! olur ki, 62!
içinde, 2 + 6 + 20 = 28 tane 3 çarpanı, 27! içinde ise, 1 + 3 + 9 = 13 tane 3 çarpanı
olacağından, 62!27! sayısının sadece 28 − 13 = 15 tane 3 çarpanı bulunur. Yani, 316
ya bölünmez.
F Wilson Teoremi :
 bir asal sayı olmak üzere
( − 1)! + 1 ≡ 0 (mod )
denkliği sağlanır. Örneğin, 12! + 1 sayısı 13’e tam bölünür.
Örnek 92 a) 30! sayısının 31’e bölümünden kalan kaçtır?
b) 11 · 12 · 12! sayısının 13’e bölümünden kalan kaçtır?
c) 10! sayısının 13’e bölümünden kalan kaçtır?
Çözüm : a) Wilson Teoremine göre,
30! ≡ −1 ≡ 30 (mod 31)
olacağından, 30! sayısının 31’e bölümünden kalan 30’dur.
b) Wilson Teoremine göre, 12! ≡ −1 (mod 13) olduğundan,
11 · 12 · 12! = 11 · 12 · (−1) ≡ 11 · (−1) (−1) ≡ 11 (mod 13)
bulunur.
c) Wilson Teoremine göre, 12! ≡ −1 (mod 13) olduğundan,
12 · 11 · 10! ≡ −1 (mod 13)
yazılabilir. Buradan, (−1) · (−2) · 10! ≡ 12 (mod 13) şeklinde yazarsak,
10! = 6 (mod 13)
elde edilir.
Alıştırmalar : a) 16! sayısının 17’ye bölümünden kalan kaçtır?
b) 15 · 16! sayısının 17’ye bölümünden kalan kaçtır?
c) 14! sayısının 17’ye bölümünden kalan kaçtır?
Yanıt : a) 16,
b) 2,
c) 8.
50
Matematik Olimpiyatlarına Hazırlık 1
1.10
Bir Sayının Tamkısmı (Tamdeğer) ve Kesir Kısmı
Bir reel sayıdan büyük olmayan en büyük tamsayıya bu reel sayının tamdeğeri
denir. Bir  reel sayısının tamdeğeri db||ce ile, geri kalan virgüllü kısmı da {} ile
gösterilir. Yani,  = db||ce + {} şeklindedir. Buradaki, {} ifadesine de  sayısının
kesir kısmı denir. {} =  − db||ce şeklinde bulunur. Örneğin,
½
½
db|7 45|ce = 7
db||ce = 3


{7 45} = 0 45
{} = 0 1415
½ §¥̄√ ¯¦¨
½
¯ 3¯ = 1
db|−6 2|ce = −7
©√ ª

{−6 2} = 0 8
3 = 0 732
gibi. db||ce =  ∈  ise,  ≤    + 1 olması gerektiğini görünüz.
Tamdeğer, sayılar teorisi bölümünde ayrıntılı olarak ele alınacaktır. Burada
sadece bazı sorularda gösterimin anlaşılabilmesi için kısaca belirtilmiştir.
Örnek 93 100! sayısı 3’ün en fazla kaçıncı kuvvetine bölünür?
Çözüm : 1’den 100’e kadar olan sayılardan,
¥̄§ 100 ¯¨¦
¯ ¯ = 33 tanesi 3’e bölünür.
3
¥̄§ 100 ¯¨¦
¯ 2 ¯ = 11 tanesi 32 ’ye bölünür.
3
¥̄§ 100 ¯¨¦
¯ 3 ¯ = 3 tanesi 33 ’e bölünür
3
¥̄§ 100 ¯¨¦
¯ 4 ¯ = 1 tanesi 34 ’e bölünür.
3
O halde,
¥̄§ 100 ¯¨¦ ¥̄§ 100 ¯¨¦ ¥̄§ 100 ¯¨¦ ¥̄§ 100 ¯¨¦
¯ 1 ¯ + ¯ 2 ¯ + ¯ 3 ¯ + ¯ 4 ¯ = 33 + 11 + 3 + 1 = 48
3
3
3
3
olduğundan, 100! sayısı 3’ün en fazla 48’inci kuvvetine bölünür.
Örnek 94 x ve y pozitif tamsayılar olmak üzere,
100!
= x7
50!
ifadesinde y en büyük kaçtır?
¥̄§ ¯¨¦ ¥̄§ ¯¨¦
¥̄§ ¯¨¦ ¥̄§ 100 ¯¨¦
¯ + ¯ 2 ¯ = 14 + 2 = 16 ve ¯ 501 ¯ + ¯ 502 ¯ = 7 + 1 = 8 olduğundan,
Çözüm : ¯ 100
1
7
7
7
7
100!
50!
ifadesinde en çok 16 − 8 = 8 tane 7 çarpanı vardır ve  en büyük 8 olur.
51
Temel Bilgiler
Örnek 95 db|3x + y|ce = 12 ve db|x + 3y|ce = 14 ise db|x + y|ce kaçtır?
Çözüm : db|3 + |ce = 12 eşitliğine göre, 12 ≤ 3 +   13 olur.
db| + 3|ce = 14 eşitliğine göre de 14 ≤  + 3  15 olur.
Bu eşitsizlikleri taraf tarafa toplarsak,
26 ≤ 4 + 4  28
olur. Bu eşitsizliğe göre,
26
≤+ 7
4
olacağından, db| + |ce = 6 bulunur.
¯¨¦
¥̄§
Örnek 96 x = ¯ 12 x− 43 ¯ + db|x|ce denklemini sağlayan kaç x reel sayısı vardır?
Çözüm : Eşitliğin sağ kısmında ifadelerin hepsi tamsayı olduğundan, sol kısmı olan
 sayısı da tamsayı olmalıdır. Bu durumda, db||ce =  olacağından,
¯¨¦
¯¨¦
¥̄§
¥̄§
 = ¯ 1  − 4 ¯ +  ise, ¯ 1  − 4 ¯ = 0
2
3
2
3
olmalıdır. Son eşitlikten de,
0≤
1
4
− 1
2
3
bulunur. Bu eşitsizlikten
8
14
≤
3
3
çıkar. Bu aralıktaki  tamsayıları  = 3 ve  = 4’tür.
¯¨¦
¥̄§
¯ = x denklemini sağlayan kaç x tamsayısı vardır?
Örnek 97 ¯ 3−7
5
3
¥̄§ 3−7 ¯¨¦

Çözüm : ¯ 5 ¯ tamsayı olduğundan, eşitliğin sağ tarafı olan de tamsayı olmalıdır
3
ve  tamsayı olduğundan , 3’ün bir katı olmalıdır. Diğer taraftan sırasıyla,
3 − 7


≤
 + 1
3
5
3
3 − 7 
0≤
−  1
5
3
4 − 21
0≤
 1
15
21
≤9
4
eşitsizlikleri elde edilir. Son eşitsizliği de sadece  = 6 tamsayısı sağlar.
52
Matematik Olimpiyatlarına Hazırlık 1
db|x|ce −1
olduğuna göre x kaçtır?
3
Çözüm : Denklemde,  = db||ce + {} yazalım. Buna göre, sırasıyla
db||ce − 1

db||ce + {} + 4 = 2 {} + 3 db||ce −
3
3 db||ce + 3 {} + 12 = 6 {} + 9 db||ce − db||ce + 1
3 {} + 5 db||ce = 11
Örnek 98 x + 4 = 2 {x} +3 db|x|ce −
elde edilir. Bu eşitliğe göre, sağ taraf tamsayı ve db||ce de tamsayı olduğundan,

 ∈ {0 1 2} olmak üzere, {} = olmalıdır. Buna göre,
3
 + 5 db||ce = 11
eşitliğinden,
db||ce =
11 − 
5
olur. Buradan da,  = 1 elde edilir. O halde, db||ce = 2 ve  = 2 +
7
1
= bulunur.
3
3
F {} = 0 eşitliği ancak ve ancak  bir tamsayıysa geçerlidir.
Örnek 99 db|4x|ce + {4x − 5} = db|4 − x|ce olduğuna göre, x kaçtır?
Çözüm : db|4|ce  db|4 − |ce ifadeleri tamsayı olduğuna göre, {4 − 5} = 0 olmalıdır.
+5
olur. Bunu
Yani, 4 − 5 tamsayı olmalıdır. 4 − 5 =  ∈ Z denilirse,  =
4
denklemde yerine yazalım. Buna göre,
¯¨¦
¥̄§
¯
db| + 5|ce + {} = ¯4 − +5
4
denkleminde,  + 5 tamsayı olduğundan, db| + 5|ce =  + 5’tir. O halde,
¯¨¦
¥̄§
¯
 + 5 = 4 + ¯− +5
4
¯¨¦
¥̄§
¯ =  + 1 elde edilir. Buna göre, sırasıyla
eşitliğinden, ¯− +5
4
+5
+1 ≤ −
  + 2
4
4 + 4 ≤ − − 5  4 + 8
4 + 9 ≤ −  4 + 13
9 ≤ −5  13
−13
−9
≥ 
5
5
+5
3
olacağından,  = −2 elde edilir. Bu durumda,  =
= olur.
4
4
53
Temel Bilgiler
Örnek 100 db|x|ce ve {x} ifadeleri, sırasıyla x gerçel sayısının tam ve kesirli kısım­
larını göstermektedir. x ∈ R+ için, db|x|ce · {x} çarpımının 13’ten küçük olmaması
için, x sayısı en az kaç olmalıdır?
Çözüm :  reel sayısı için,
 = db||ce + {}
ifadesinde, {} kesir kısmı 1’den küçük bir sayı olacağından, db||ce ile {}çarpıldığında,
db||ce değerinden daha küçük bir değer elde edilecektir. O halde, db||ce · {} çarpımının
13’ten büyük veya eşit olabilmesi için, db||ce sayısı en az 14 olmalıdır. Buna göre, en
küçük  değeri için, db||ce · {} çarpımını 13 alabiliriz. Bu durumda,
13
209
 = 14 +
=
14
14
sayısı, istenen koşulu sağlayan en küçük  reel sayısıdır.
54
Matematik Olimpiyatlarına Hazırlık 1
1.11
Mutlak Değer
Bir tamsayının mutlak değeri, o sayının sayı doğrusunda bulunduğu noktanın
başlangıç noktasına uzaklığıdır. Bir  sayısının mutlak değeri, || ile gösterilir.
½

 ≥ 0 ise
|| =
−
  0 ise
şeklinde yazılabilir. Mutlak değerin aşağıdaki özelliklerini inceleyiniz.
1.
2.
3.
4.
5.
6.
7.
|−| = ||
 ∈ R+ olmak üzere, || =  ise  = − veya  =  olur.
 ∈ R+ olmak üzere, ||   ise −     olur.
 ∈ R+ olmak üzere, ||   ise   − veya    olur.
|| || = ||  ¯ ¯
|| ¯¯  ¯¯
 6= 0 için,
=

|| ¯  ¯
 ∈  için, | | = || .
Örnek 101 |x − 5| + |3 − x| = 2 denkleminin kaç tamsayı çözümü vardır?
Çözüm : Denklemi aşağıdaki üç durum için inceleyeceğiz.
i)  ≤ 3
ii) 3    5
iii) 5 ≤ 
Bu üç duruma teker teker bakalım.
i)  ≤ 3 ise, − + 5 + 3 −  = 2 eşitliğinden  = 3 çözümü elde edilir.
ii) 3    5 ise, − + 5 − 3 +  = 2, yani 2 = 2 olduğundan, 3    5
aralığındaki her reel sayı bir çözümdür.
iii) 5 ≤  ise,  − 5 − 3 +  = 2 eşitliğinden,  = 5 çözümü elde edilir.
Denklemin tamsayı çözümlerini aradığımızdan,  = 3  = 5 ve  = 4 çözüm olur.
Örnek 102
|x − 2008| −100
≤ 0 eşitsizliğini sağlayan kaç x tam­
|x2 −2009x| + |x − 2009|
sayısı vardır?
¯
¯
Çözüm :  = 2009 hariç, ¯2 − 2009¯ + | − 2009|  0 olduğundan,
| − 2008| − 100 ≤ 0
sağlanmalıdır.  = 2009 için payda sıfır olacağından,  = 2009 olamayacağı unutul­
mamalıdır. Buna göre,
| − 2008| ≤ 100
eşitsizliğinden, −100 ≤  − 2008 ≤ 100 yani, 1908 ≤  ≤ 2108 olur. Bu aralıkta,
2107 − 1907 + 1 = 201 tamsayı vardır. Fakat, 2009 sağlamadığından, 1 çıkarılırsa
200 tamsayının bu eşitsizliği sağladığı görülür.
55
Temel Bilgiler
Örnek 103 8 < |x − 3| < 13 eşitsizliğini sağlayan kaç tane x tamsayısı vardır?
Çözüm : 8  | − 3|  13 eşitsizliğinin sağlanması için,
8  − + 3  13 veya 8   − 3  13
olmalıdır. Bu eşitsizliklerden sırasıyla
−10    −5 veya 11    16
elde edilir. Böylece çözüm kümesi,
{ : −10    −5 veya 11    16}
bulunur. O halde eşitsizliği sağlayan tamsayılar,
−9 −8 −7 −6 12 13 14 ve 15
sayılarıdır.
Örnek 104 4x + 5y = 7 denklemini sağlayan x, y tamsayıları için, 5 |x|−3|y|
ifadesinin en küçük değeri kaç olur?
Çözüm : Denklemi sağlayan tamsayıların farklı işaretli olacağı aşikardır. Buna göre,
i)   0 ve   0 olursa,
µ
¶
7 − 4
1 − 2
5 || − 3 || = −5 − 3 = −5 − 3
= −3 − 4 −
5
5
olur. Bu ifade en küçük  = −2 için, 1 değerini alır.
ii)   0 ve   0 olursa,
µ
¶
7 − 4
1 + 3
5 || − 3 || = 5 + 3 = 5 + 3
= 2 + 4 +
5
5
olur ve bu ifade en küçük  = 3 için, 12 değerini alır. O halde en küçük değeri 1
bulunur.
¯¯¯¯¯
¯
¯
¯
¯
¯
Örnek 105 ¯¯¯¯¯x2 +3x − 5¯ −4¯ −3¯ −2¯ −1¯ = 3x − 15 denkleminin kaç
çözümü vardır?
Çözüm : Eşitliğin sol tarafı pozitif olduğundan sağ tarafı da pozitif olmalıdır. Buna
göre, 3 − 15  0 eşitsizliğinden   5 olur. O halde, eşitliğin sol tarafında
2 + 3  25 + 15 = 40
olduğundan, mutlak değerlerin tümünün içi pozitif olacaktır. Yani sol taraftaki mutlak
değerlerin tümünü kaldırabiliriz. 2 + 3 − 15 = 3 − 15 eşitliğinden, 2 = 0,
buradan da  = 0 elde edilir, ki   5 olması gerektiğinden, bu bir çözüm değildir.
Yani denklemin kökü yoktur.
56
Örnek 106
Matematik Olimpiyatlarına Hazırlık 1
½
|x + y| = 1
denklem sisteminin çözüm kümesini bulunuz.
|x| + |y| = 1
Çözüm : Dört ayrı durum için denklemleri inceleyelim.
i)  ≤ 0 ve  ≤ 0 olması durumunda, denklemler
½
− −  = 1
− −  = 1
olacaktır. Bu denklem sistemi + = −1 denklemine denktir. Bu bir doğrunun denk­
lemidir ve bu doğru üzerindeki her nokta verilen denklem sisteminin bir çözümüdür
ii)  ≤ 0 ve  ≥ 0 olması durumunda, denklemler,
½
| + | = 1
− +  = 1
olacaktır. İkinci denklemde  = 1 +  değerini, birinci denklemde yazarsak,
| + 1 + | = 1
olur. Bu eşitlikten de, 2 + 1 = −1 veya 2 + 1 = 1 elde edilir, yani
( ) = (−1 0) veya ( ) = (0 1)
olmalıdır.
iii)  ≥ 0 ve  ≤ 0 olması durumunda,
 = 1 +  ve | + 1 + | = 1
olacak ve bu denklemin çözümünden de, 2 + 1 = −1 veya 2 + 1 = 1 yani,
( ) = (0 −1) veya ( ) = (1 0)
elde edilecektir.
iv )  ≥ 0 ve  ≥ 0 olması durumunda, denklemler
½
+ =1
+ =1
olacaktır. Bu sistemden de,  +  = 1 elde edilir.
O halde, denklem sisteminin çözüm kümesi,
 = {( ) :    0 ve  +  = 1} ;
 = {( ) :    0 ve  +  = −1} ;
 = {(−1 0)  (0 1)  (0 −1)  (1 0)}
olmak üzere,  ∪  ∪  olarak bulunur.
F | − | + | + | =  gibi, simetrik mutlak değerli denklemlerde, 0 bir çözümse
−0 ’da bir çözümdür.
Örneğin, | − 2| + | + 2| = 6 denklemini hem  = 3 hem de  = −3 sağlar.
57
Temel Bilgiler
Örnek 107 |x − 10|+|x − 9|+· · ·+|x − 1|+|x|+|x+1|+· · ·+|x+10| =c
denkleminin tek çözümü olduğunu biliyoruz. Buna göre, c sayısını bulunuz.
Çözüm : 0 verilen denklemin bir kökü olsun, bu durumda −0 da denklemi sağladı­
ğından denklemin bir köküdür. Denklemin tek kökü olduğuna göre, 0 = −0
olmalıdır. Buradan, 20 = 0 eşitliğinden 0 = 0 olur. Bu değeri denklemde yazarsak,
bulunur.
 = 10 + 9 + · · · 1 + 0 + 1 + · · · + 9 + 10
10 · 11
= 2 (1 + · · · + 9 + 10) = 2
= 110
2
18
ifadesinin en büyük değeri kaçtır?
|x − 1| + |x − 5| + |x − 13|
Çözüm : Bu ifadenin en büyük değeri için, payda en küçük olmalıdır. Paydadaki
mutlak değerli ifadeleri 0 yapan değerleri sayı doğrusu üzerinde gösterelim.
Örnek 108
1
A
13
C
5
B
Şimdi bu sayı doğrusu üzerinde öyle bir  noktası seçeceğiz ki, payda bu  değeri için
en küçük olsun. Seçeceğimiz bu nokta, B haricinde bir nokta olursa, bulunan uzunluk
[] uzunluğundan büyük olacaktır. O halde, seçmemiz gereken nokta B noktası,
yani  = 5 değeri olmalıdır. Böylece, en büyük değer,
18
3
=
4+0+8
2
bulunur.
F | + | ≤ || + || eşitsizliği sağlanır. Bu eşitsizliğe üçgen eşitsizliği de denir.
Kanıt : Her   ∈ R için,
2
2
 ≤ || ve 2 +  2 = || + ||
yazılabilir. Buna göre,
2
2
2 + 2 +  2 ≤ || + 2 || + ||
eşitsizliğinden, ( + )2 ≤ (|| + ||)2 elde edilir. Her iki tarafın karekökü alınırsa,
| + | ≤ || + ||
olduğu görülür.
58
Matematik Olimpiyatlarına Hazırlık 1
1.12
Üslü ve Köklü Sayılar
Üslü ve köklü sayılarla ilgili belli başlı tanımları verip, üslü ve köklü sayıların
özelliklerini kısaca hatırlayalım.
 tane
z
}|
{
I  ∈ N olmak üzere,  =  ·  ·  · · ·  ifadesinde  sayısına taban ve  sayısına
da üs veya kuvvet denir.  6= 0 için, 0 = 1 biçiminde tanımlanır. 00 sayısını
tanımlamayacağız. Bu ifadeyi tanımsız olarak kabul edeceğiz.
I   ∈ N için,
 ³  ´


i) ( ) = ·  ii) ·  = +  iii) ·  = ( · ) , iv)  =


eşitlikleri sağlanır.

I   ∈ N için,  = − biçiminde tanımlanır. Buna göre,  = 0 alınırsa,

1
− =  eşitliği elde edilir.

I  =  eşitliğinin sağlanması için gerek ve yeter şart  =  olmasıdır.
√
I  ∈ Z+ olmak üzere,   sayısına,
√  sayısının ’inci dereceden kökü denir. 
sayısının çift sayı olması durumunda,   ifadesinde  negatif olamaz.

√
I  ∈ Z+ ve  ∈ Z ve  ∈ R+ için,   =   biçiminde tanımlanır. Böylece,
rasyonel üslü ifadeleri tanımlamış oluruz. Tamsayı üslü sayılardaki özellikler, ras­
yonel üslü sayılarda da geçerlidir.
√
√ 
I  ∈ Z+ ve  ∈ Z için, (  ) =   eşitliği sağlanır.
√
I  ∈ Z+ için,   sayısının değeri  sayısının tek ve çift olmasına göre değişir.
½
√

 tek sayı ise,

 =
||
 çift sayı ise

biçiminde tanımlanır.
I   ∈ Z+ olmak üzere,
r
√

√
√
p√
√
√
√
√



·





·

= 
 =

=

   =  ·  √
ve    =



eşitlikleri sağlanır.
√
√
√
I  ∈ Z+ için,      ise         biçiminde sıralanır.
I  ∈ R için, 1 = 1 olarak tanımlanır. Ayrıca,
½
−1
 tek ise,

(−1) =
1
 çift ise,
√

·
biçiminde tanımlanır.
I Buna göre,  = 1 olması için gerek ve yeter koşul, i)  6= 0 ve  ∈ R ve
 = 0, ii)  = 1 ve  ∈ R iii)  = −1 ve  çift tamsayı, durumlarından birinin
sağlanmasıdır.
59
Temel Bilgiler
Paydayı Rasyonel Yapma
  ∈ R ve  6= 0 olmak üzere,  biçiminde ifadeler bir rasyonel sayı
olmak zorunda değildir. Bu nedenle bu tür ifadelere kesir demek doğru olmaz. Fakat,
genel olarak  sayısının köklü bir sayı olması istenmez. Bunun için,  ifadesi ile
çarpıldığında bir rasyonel sayı elde edilecek şekilde, hem  hem de  sayısı uygun
bir reel sayıyla çarpılabilir. Şimdi, verilen bir  sayısına göre,  sayısını rasyonel
yapacak uygun çarpanları verelim.
√


1.     6= 0 için, √
şeklindeki ifadeler − ile çarpılır.



√
√

2
√
 + 2  ile çarpılır.
2. 2
√ şeklindeki ifadeler
2
−

√
√

2
√
 − 2  ile çarpılır.
3. 2
√ şeklindeki ifadeler
2
+

√
√
√

3
3
şeklindeki ifadeler 2 + 3  + 2 ile çarpılır.
4. √
√
3
3
− 
√
√
√

3
3
şeklindeki ifadeler 2 − 3  + 2 ile çarpılır.
5. √
√
3
3
+ 
2
Örnek 109 (x − 3)2
−18
= 1 denkleminin kaç kökü vardır?
Çözüm : i)  − 3 = 1 ve 22 − 18 ∈ R eşitliğinden  = 4 olur.
ii)  − 3 6= 0 ve 22 − 18 = 0 ise 2 ( − 3) ( + 3) = 0 eşitliğinden  = 3
veya  = −3 olur. Fakat  6= 3 olduğundan  = −3 olabilir.
iii)  − 3 = −1 ve 22 − 18 çift olabilir.  = 2 olur ve  = 2 için, 22 − 18
sayısı çift olduğundan  = 2 de bir çözümdür. O halde 3 çözüm vardır.
Örnek 110 5 · 22 −36 · 2 +32 −2 · 2 ·3 +81 = 0 olduğuna göre x ve y
arasında daha basit bir bağıntı bulunuz.
Çözüm : 5 · 22 − 36 · 2 + 32 − 2 · 2 · 3 + 81 = 0 eşitliğini,
¡ 2
¢
¢ ¡
2 − 2 · 2 · 3 + 32 + 22(+1) − 36 · 2 + 81 = 0
şeklinde yazarsak,
¡
¢2
2
(2 − 3 ) + 2+1 − 9 = 0
bulunur. Buradan, 2 = 3 ve 2+1 = 9 elde edilir. Sonuncu eşitliklerin ikincisinden
elde edilen 3 = 2(+1)2 ifadesi birinci eşitlikte yazılırsa, 2 = 2(+1)2 eşitliğinden
 = 2 ( + 1) elde edilir.
60
Matematik Olimpiyatlarına Hazırlık 1
√
√
√
Örnek 111
2x + 3 + 4 3 − x + 3  + 2 ifadesi bir reel sayı olacak şekilde kaç
x tamsayısı vardır?
Çözüm : 2 + 3 ≥ 0 ve 3 −  ≥ 0 olmalıdır. Buna göre,
−32 ≤  ≤ 3
olacağından, bu aralıktaki, tamsayılar −1 0 1 2 ve 3 olur.
Örnek 112 a, b ve c rasyonel sayılar olmak üzere,
√
a
3
5
2
√
√ =√ −
−
b
c
1− 5 − 6
30
eşitliği sağlanıyor ise a, b ve c’yi bulunuz.
√
√ ¢ √
¡
2

5 3
√
√ = √ −
Çözüm :
− eşitliğinin sol tarafını 1 − 5 + 6


1 − √5 −√ 6
30
√
((1− 5)+ 6)
( 30)
ile çarpalım. Buradan, sırasıyla,
√ ¢
√
¡
√
√
2 1− 5+ 6
 30
5 3
√
=
−
− 
30


−2 5
√
√
√
√
5  6 3 5
1− 5+ 6 =
−
+

6

√
√
elde edilir. Son eşitliğe göre, 5, 6 ve 5 sayıları birbirinden bağımsız olduklarından,
3

5
= 1 −1 =
ve 1 = −


6
olacağından,  = 5  = −3 ve  = −6 bulunur.
F İç İçe Kökler
p
p
√
√
 + 2  veya  − 2  şeklindeki ifadelerde, kök içerisindeki ifadenin bir
reel sayının karesi olabilmesi için,
 =  +  ve  =  · 
olacak şekilde  ve  sayılarının olması yeterlidir. Bu şekilde  ve  sayıları var ise,
r³
p
√ √ ´2 √ √
√
+2  =
+  = + 
↓
↓
+ · r
³√ √ ´2 ¯√ √ ¯
p
√
¯
¯
−2  =
−  = ¯ − ¯
↓
↓
+ ·
şeklinde dışarı çıkacaktır.
Temel Bilgiler
Örnek 113
61
p √
p √
4− 7 − 4+ 7 =?
Çözüm : Önce iç içe kök olan bu ifadeleri kuralımızı uygulayabilecek şekile sokmaya
çalışalım. Bunun için, kökün içindeki ifadeyi 2 ile çarpıp bölüyoruz.
r
√
p
√
√
1 p
8−2 7
4− 7=
= √ ( 8 − 2 7)
2
2
p
¯√
√
√ ¯
eşitliğinde, 8 − 2 7 = ¯ 7 − 1¯ olduğundan,
√
√
p
√
7− 1
√
4− 7=
2
√
√
p
√
7+ 1
√
olur. Benzer düşünceyle, 4 + 7 =
bulunur. Böylece,
2
√
√
p
p
√
√
√
−2
7−1
7+1
4− 7− 4+ 7= √
− √
= √ =− 2
2
2
2
elde edilir.
Örnek 114 0 < x < 1 olmak üzere,
q
q
√
√
2
3
x +2 x +x − x2 −2 x3 +x = 5
denkleminin kaç reel kökü vardır?
¯√
p
p
√
√
√
√
√ ¯¯
¯
Çözüm : ( 2 +  + 2 3 ) = 2 +  ve 2 − 2 3 +  = ¯ 2 − ¯
↓
↓
2 +  2 · 
¯√
√ ¯¯
√
¯
eşitliklerinde, 0    1 iken, 2   olduğundan, ¯ 2 − ¯ =  −  olur.
Böylece,
p
p
√
√
2 + 2 3 +  − 2 − 2 3 +  = 5
denkleminde bu eşitlikleri kullanırsak,
√
√
+ − +=5
olur. Bu denklemden de, 2 = 5 ve  = 52 olur. Fakat, 0    1 olduğundan, bu
mümkün değildir. O halde denklemin kökü yoktur.
√ ¢32 ¡
√ ¢32
¡
Örnek 115 52 + 6 43
− 52 − 6 43
=? (AIME 1990)
p
p
√
√
√
√
√
Çözüm : 52 ± 6 43 = 52 ± 2 43 · 9 = 43 ± 9 = 43 ± 3 olduğundan,
´3 ³√
´3
³
√ ´32 ³√
√ ´32 ³
− 52 − 6 43
=
43 + 3 −
43 − 3
52 + 6 43
√
√
eşitliğinde, 43 + 3 =  ve 43 − 3 =  diyelim.
62
Matematik Olimpiyatlarına Hazırlık 1
Bu durumda,
³√
´3 ³√
´3
¡
¢
43 + 3 −
43 − 3 = 3 −  3 = ( − ) 2 +  +  2
eşitliğinde  ve  yerine yazılırsa,
³√
´3 ³√
´3
³
´
√
√
43 + 3 −
43 − 3 = 6 43 + 6 43 + 9 + 43 − 9 + 43 − 6 43 + 9
¢3 ¡√
¢3
¡√
43 + 3 − 43 − 3 = 828 elde edilir.
eşitliğinden
√
√
2+ 3
2− 3
p
p
+
√
√
√
√ =?
2 + 2+ 3
2 − 2− 3
p
√
√
p
√
4±2 3
3±1
√
= √
olduğundan,
Çözüm : 2 ± 3 =
2
2
√
√
√
√
2− 3
2+ 3
2− 3
2+ 3
√
√
p
p
+
√ +√
√ =
√
√
√
3+1
3−1
2+ 2+ 3
2− 2− 3
2+ √
2− √
2
2
Ã
√
√
√
√ !
√
√
2 2+ 6 2 2− 6 √
2+ 3 2− 3
√ +
√
√ +
√
=
= 2
3+ 3
3− 3
3+ 3 3− 3
Ã
√ !
√
√
√
√ 2+ 3 2− 3
√
√
3+ 3+3− 3
√ +
√ )= 2
= 2(
= 2
9−3
3 +√ 3 3 −√ 3
(3− 3)
(3+ 3)
bulunur.
Örnek 116
√
√
31
3 − 2 ve
sayılarından hangisi daha büyüktür?
100
√
√
31
olduğunu kabul edelim. Her iki tarafın karesini alırsak,
Çözüm : 3 − 2 
100
√
961
5−2 6 4
10
√
961
49 039
yani 5 −
olduğundan,
 2 6 olur. Sol taraf
10000
104
3
√
49 · 10
2 6
4
10
yazılabilir. Tekrar kare alırsak,
2401
 24
102
√
√
31
eşitsizliği elde edilir ki, bu yanlıştır. O halde, 3 − 2 
kabulümüz yanlıştır.
100
√
√
Yani, 3 − 2  31100 olur.
Örnek 117
63
Temel Bilgiler
Örnek 118 2, 3, 4, ..., 1000 sayılarından kaçı küpkökünden küçük olan en büyük
tamsayıya bölünür?
Çözüm :
√
√
√
3
2 3 3  3 8 sayılarının küpkökünden küçük olan en büyük tamsayı 1,
√
√
√
3
9 3 10  3 27 sayılarının küpkökünden küçük olan en büyük tamsayı 2,
√
√
√
3
28 3 29  3 64 sayılarının küpkökünden küçük olan en büyük tamsayı 3,
..
.
√
√
√
3
513 3 514  3 729 sayılarının küpkökünden küçük olan en büyük tamsayı 8,
√
√
√
3
730 3 731  3 1000 sayılarının küpkökünden küçük olan en büyük tamsayı 9’dur.
En genel halde,
3 + 1 3 + 2  ( + 1)3
sayılarının köp kökünden küçük olan en büyük sayı ’dir. Burada,
¡
¢
( + 1)3 − 3 + 1 + 1 = 32 + 3 + 1
sayı olduğundan, bu 32 + 3 + 1 sayıdan,   1 için  ile bölünenler :
¡
¢
3 +  3 + 2  3 + 32 + 3
sayılarıdır.  = 1 ise bu sayıların tamamı 1’e bölüneceğinden koşulu sağlarlar ve
bunların sayısı 7 tanedir. Fakat,   1 için,  ile bölünenlerin sayısı, artış miktarı 
olduğundan,
¡
¢ ¡
¢
3 + 32 + 3 − 3 + 
Son Terim − İlk Terim
+1 =
+1
Artış Miktarı

= 3 + 3
tanedir. Buna göre,  = 1 için 7 ve  = 2 3  9 için 3 + 3 değerlerini toplarsak
istenen elde edilir.
1 + (3·1 + 3) + (3·2 + 3) + (3·3 + 3) + · · · + (3·9 + 3)
toplamını,
şeklinde yazarsak,
1 + 3 (1 + 2 + · · · + 9) + (3 + 3 + · · · + 3)
1+3·
buluruz.
9 · 10
+ 27 = 163
2
64
Matematik Olimpiyatlarına Hazırlık 1
1.13
Oran ve Orantı
En az biri sıfırdan farklı olan aynı birimdeki iki çokluğun bölümüne oran denir.
İki veya daha fazla oranın eşitliğine de orantı denilir.


= =


orantısında  sayısına orantı sabiti denir. Orantının bazı özelliklerini hatırlayalım.



orantısı  :  :  =  :  :  şeklinde gösterilir.
* = =



* Bir orantıda, payların toplamı paydaların toplamına oranlandığında orantı sabiti
değişmez.



++
Eğer = = =  ise,
=  olur.



++


 +  + 

=
=
=  ise,
=  olur.
Eğer



 +  + 
* Eğer


 + 
 + 
 + 

= =
ise,
=
=
olur.



 + 
 + 
 + 
* İki çokluktan biri artarken diğeri de artıyorsa bu iki çokluk doğru orantılı, biri
artarken diğeri azalıyorsa bu iki çokluk ters orantılıdır denir.  ve  doğru

=   ve  ters orantılı ise,  =  olur.
orantılı ise,

5x − y
2a
x + 3y
2b
=
ve
=
olduğuna
x − 7y + 4
5a − 3b
x − 7y + 4
5a − 3b
göre, x kaçtır?
Çözüm : Birinci denklem 5 ile, ikinci denklem de −3 ile çarpılıp taraf tarafa toplanırsa,
10 − 6
25 − 5 − 3 − 9
=
=2
 − 7 + 4
5 − 3
eşitliğinden, 25 − 5 − 3 − 9 = 2 − 14 + 8 olur. Bu denklemin çözümünden,
 = 25 elde edilir.
Örnek 119
Örnek 120 Sıfırdan ve birbirinden farklı a, b ve c sayıları için,
b+c
c+a
a+b
=
=
=k
c
a
b
ise, k kaçtır?
+
+
+
+1=
+1=
+ 1 =  + 1 eşitliklerinden,
Çözüm :



++
++
++
=
=
=+1



olur.   ve  birbirinden farklı olduğundan bu eşitliğin sağlanması için  +  +  = 0
olmalıdır. Bu durumda,  = −1 bulunur.
65
Temel Bilgiler
1.14
Karışık Örnekler
Örnek 121 2, 56, 2, 61, 2, 65, 2, 71, 2, 79, 2, 82, 2, 86 sayılarının
her birini bir tamsayı değerine yuvarlanarak yapılan toplama işlemindeki toplam,
gerçek toplama eşit olsun. Her bir yuvarlamadaki hataların en büyük olanı E
olsun. E sayısının en küçük değeri için, 100E sayısı kaçtır? (AIME 1985)
Çözüm : 7 sayının toplamı hesaplanırsa,
2 56 + 2 61 + 2 65 + 2 71 + 2 79 + 2 82 + 2 86 = 19
bulunur. Açıktır ki, yuvarlamadan sonra yapılan toplamanın en küçük olması için,
sayıların 2 ya da 3’e yuvarlanması gerekir. Sayıların 2 ve 3’e yuvarlanmasında hata
  1 olurken, aksi halde   1 olacaktır. Böylece, beş tane 3 ve iki tane 2 olması
gerektiği görülür. Yuvarlamalardaki en küçük hata yapılması için verilen sayılardan
en küçük olan ikisini 2 için düşünmeliyiz. Böylece, yuvarlamalarda yapılan hatalar
sırasıyla,
0 56, 0 61, 0 35, 0 29, 0 21, 0 18, 0 14
olacaktır. Bu hataların en büyüğü  = 0 61 ve 100 = 61 elde edilir.
Örnek 122 |x+ |x| +a| + |x− |x| −a| = 2 denkleminin tam üç çözümü olacak
şekilde kaç a sayısı vardır?
Çözüm :  denklemin bir çözümü olsun. Denklemde (−) yazalım.
|(−) + |(−)| + | + |(−) − |(−)| − | = | − || − | + | + || + | = 2
olduğundan, (−) de bir çözümdür. Denklemin üç tane, yani tek sayıda çözümü
olması isteniyor. Bunun için,  = 0 denklemin köklerinden biri olmalıdır. Denklemde
 = 0 yazarsak,
|| + |−| = 2
2 || = 2
eşitliğine göre  = ±1 olmalıdır.
 = 1 ise,  ≥ 0 için, 2 + 2 = 2 denkleminden,  = 0 ve  çözüm iken (−)
de bir çözüm olduğundan sadece 1 çözümü olduğu görülür. O halde,  = 1 olamaz.
 = −1 ise,  ≥ 0 ise, |2 − 1| + 1 = 2 ise |2 − 1| = 1 olur. Bu eşitlikten
de,
½
2 − 1 = 1
2 − 1 = −1
olmalıdır. Yani,  = 0 ve  = 1 olur. (−)  yani, −1 de denklemin bir çözümü
olacağından,  = −1 için denklemin 3 çözümü olduğu görülür.
66
Matematik Olimpiyatlarına Hazırlık 1
p
√
Örnek 123 x = 32007 olmak üzere, x2 +2x + 4 ve 4x2 +2x + 1 sayıları
arasında kaç tamsayı vardır?
√
√
Çözüm :  + 1  2 + 2 + 4   + 2 ve 2  42 + 2 + 1  2 + 1
√
√
olduğundan, 2 + 2 + 4 ve 42 + 2 + 1 sayıları arasındaki tamsayı sayısı
bulunur.
2 − ( + 2) + 1 =  − 1
√
Örnek 124 1 ≤ a ≤ 100 ve 1 ≤ b ≤ 100 olmak üzere, a+ b +
ifadesi tamsayı olacak şekilde, kaç (a, b) tamsayı çifti vardır?
1
√
a+ b
Çözüm : Verilen ifadeyi aşağıdaki gibi düzenleyelim.
√
√
1
 +  + 1
√ =+
√
 = + +
+ 
+ 
³√
´
 +   − 2 +  + 1
 + 1 − 2
√
√
= +
= 2 +
+ 
+ 
Bu eşitliğe göre, 2 =  + 1 ise  ifadesi tamsayı olur. 2 6=  + 1 olduğunda 
tamsayı olamaz, çünkü, bu durumda
 + 1 − 2
√ =∈Z
+ 
olması gerekir. Buradan,
³
√ ´
  +  =  + 1 − 2
eşitliğine göre,
√
 + 1 − 2 − 
=

√
olacağından son eşitliğin sağ tarafı rasyonel sayı olduğundan, sol taraftaki√  sayısı
da bir rasyonel sayı olmalıdır. Bu durumda  tamkare olmalıdır. Yani,  +  tamsayı
√
1
√ ifadesi ve dolayısıyla da
olur. Fakat,  +  ≥ 2 olduğundan,
+ 
√
1
√
+ +
+ 
ifadesi tamsayı olamaz. O halde, sadece 2 =  + 1 olması durumunda  ifadesi
tamsayı olabilir.  = 2 − 1 eşitliğine göre,
 ∈ {2 3  10}
olabileceğinden, istenen şekilde 9 sayı vardır.
67
Temel Bilgiler
Örnek 125 6! = 8 · 9 · 10 eşitliğinde 6! sayısı ardışık üç sayının çarpımı şek­
linde yazılabilmektedir. n! sayısı (n − 3) tane ardışık sayının çarpımına eşit olacak
şekilde yazılabiliyorsa, en büyük n sayısı kaç olabilir? (AIME 1990)
Çözüm : ( − 3) tane ardışık sayının çarpımı  bir pozitif tamsayı olmak üzere,
( − 3 + )!
!
şeklinde yazılabilir. Buna göre,
( − 3 + )!
! =
!
eşitliğinin sağlanması için,   3 olmalıdır. Burada, ( − 3 + )!  ! olacağın­
dan,
! ( + 1) ( + 2) · · · ( − 3 + )
= !
!
yazılabilir. Bu eşitliğe göre,
( + 1) ( + 2) · · · ( − 3 + ) = !
olmalıdır.
 = 4 alınırsa,  + 1 = 4! = 24 denklemine göre  = 23 elde edilir. Biz
en büyük  sayısını arıyoruz., bu nedenle ’nin daha büyük değeri için bu eşitliğin
sağlanıp sağlanmadığını görelim.
 = 5 alınırsa, ( + 1) ( + 2) = 120 eşitliğinden, 10    11 olur.
 = 6 alınırsa, ( + 1) ( + 2) ( + 3) = 6! eşitliğinden de 7    8 olur.
√
Yani, ’nin yaklaşık değeri −3 ! olacak ve  arttıkça  küçülecektir. O halde
istenen şekildeki en büyük  tamsayısı 23 bulunur.
Örnek 126 Aralarındaki fark 60 olan iki pozitif tamsayının karekökleri toplamı
tamkare olmayan bir pozitif tamsayının kareköküne eşit olduğuna göre, bu iki tam­
sayının toplamının alacağı maksimum değer kaçtır? (AIME 2003)
√
√
√
Çözüm : Sayılarımız  ve 60 +  olsun.  +  + 60 =  eşitliğinin her iki
tarafının karesi alınırsa,
√
2 + 60 + 2 2 + 60 = 
√
olur. Bu eşitliğe göre, 2 + 60 bir tamsayı olmalıdır. O halde, bir  ∈ Z için,
2 + 60 = 2 yazılabilir. Buradan,
olur.
2 + 60 + 900 − 2 = 900
2
( + 30) − 2 = 900
( + 30 − ) ( + 30 + ) = 900
68
Matematik Olimpiyatlarına Hazırlık 1
Son eşitlikte, her iki çarpımda çift olmalıdır. Soruda bu sayıların toplamının yani,
2 + 60’ın alabileceği en büyük değer soruluyor. Bunun için  en büyük seçilmeli, bu
nedenle  + 30 +  çarpanını maksimum seçmeliyiz.
 + 30 +  = 450 ve  + 30 −  = 2
seçilirse, 2 + 60 = 452 denkleminden  = 196 bulunur. Fakat bu durumda  bir
tamkare olduğundan istenen şekilde değildir.
 + 30 +  = 150 ve  + 30 −  = 6
seçilirse, 2 + 60 = 156 denkleminden  = 48 olur ve tamkare değildir. O halde bu
iki tamsayının toplamının alacağı maksimum değer 156 olur.
Örnek 127 2 +1 = m + 2n (m − 1) eşitliğini sağlayan kaç (k, m, n) pozitif
tamsayı üçlüsü vardır?
Çözüm :  + 2 ( − 1) = ( − 1) (2 + 1) + 1 şeklinde yazalım. Bu du­
rumda, 2 = ( − 1) (2 + 1) olur. Bu yazılışta, (2 + 1) sayısının bir tek sayı
olduğu göz önüne alınırsa,  pozitif tamsayı olmak üzere 2 + 1 formundaki bir
sayıyı ( − 1) (2 + 1) + 1 şeklinde yazmak mümkün değildir. Çünkü, 2 + 1 =
( − 1) (2 + 1) + 1 eşitliğinden,
2 = ( − 1) (2 + 1)
olur. Sol taraf 2’nin kuvveti iken sağ tarafta bir tek çarpan olamaz. O halde, bu
denklemin çözümü yoktur.
Örnek 128 x ve y pozitif tamsayılar olmak üzere, x2 +y ve x + y 2 sayılarının
her ikisinin birden tamkare olamayacağını gösteriniz. (SSCB M.O. 1966)
Çözüm : 2 sayısından büyük olan en küçük tamkare ( + 1)2 = 2 + 2 + 1
olduğundan, 2 +  tamkare ise,   2 olmalıdır. Benzer şekilde,  2 sayısından
2
büyük olan en küçük tamkare ( + 1) =  2 + 2 + 1 olduğundan,  +  2 tamkare ise
  2 olmalıdır. Fakat,   2 ve   2 eşitsizliklerinin aynı anda sağlanması
imkansızdır. O halde 2 +  ve  +  2 sayılarının her ikisi de tamkare olamaz.
2
Örnek 129 x, y ∈ R , y ≥ 0 ve y (y + 1) ≤ (x + 1) olduğuna göre,
olduğunu gösteriniz.
y (y − 1) ≤ x2
Çözüm : İspatımızı, 0 ≤  ≤ 1 ve   1 için ayrı ayrı yapalım.
i) 0 ≤  ≤ 1 olursa,  ( − 1) ≤ 0 ≤ 2 olduğundan eşitsizlik sağlanır.
69
Temel Bilgiler
1
1
ii)   1 olması durumundaysa,  +   veya  + ≤  olmasına göre
2
2
inceleyelim.
1
 +   ise,
2
µ
¶µ
¶
1
1
1
+
  ( − 1)
 −   − 1 ve  −
2
2
2
olacağından,
 ( − 1) ≤ 2 −
1
 2
4
elde edilir. Benzer şekilde,
1
 + ≤  ise,
2
µ
¶
1
= 2
 ( − 1) =  ( + 1) − 2 ≤ ( + 1) − 2  +
2
2
bulunur.
³ √
´3
p √
Örnek 130
2 3 2 +1− 12 3 2−15 = 32 olduğunu ispatlayınız.
√
√
√
√
3
3
3
2
5
2
Çözüm : 3 2 =  diyelim,
√ 32 = 2 =2 2 2 = 2 olacağından, denklemde
yerine yazılırsa, 2 + 1 − 12 − 15 = 2 olur. Kökü yalnız bırakırsak,
√
12 − 15 = −22 + 2 + 1
olur. Bu eşitliğin doğruluğunu gösterirsek ispat biter.
Her iki tarafın karesini alırsak,
¡
¢2
12 − 15 = −22 + 2 + 1 = 4 − 83 + 44 + 1
olur. Bu denklemde, 3 = 2 ve 4 = 2 olduğu kullanılırsa,
12 − 15 = 4 − 16 + 8 + 1 = 12 − 15
eşitliği elde edilir. Yani, eşitlik doğrudur.
Örnek 131 p bir asal sayı ve x > 0 n ≥ 0 tamsayılar olmak üzere, n2 ·p < 1000
eşitsizliğini sağlıyorsa,
x
2
n2 +100· = (n + x)
p
denkleminin kaç (x, n, p) çözüm üçlüsü vardır? (AÜMO 2009)
Çözüm : 2 ·  + 100 = 2 ·  + 2 + 2  eşitliğinden 100 =  · (2 + ) elde
edilir. Bu denkleme göre,  asal sayısının 2 veya 5 olabileceğini görürüz. Bu değerler
için denklemi inceleyelim.
70
Matematik Olimpiyatlarına Hazırlık 1
 = 2 için, 2 +  = 50 denkleminin çözümlerini bulalım. 2 2  1000 ise,
  500 ve buradan da  ≤ 22 olacağından, istenen şekilde 23 tane negatif olmayan
 tamsayısı ve dolayısıyla da 23 tane ( ) ikilisinin olduğu görülür.
 = 5 için, 20 = 2 +  denklemi elde edilir. Buradan   0 koşulundan dolayı,
 ≤ 9 olması gerekir. 0 ≤  ≤ 9 sayıları, problemin 2 · 5  1000 yani, 2  200
koşulunu da sağlarlar. O halde,  = 5 durumunda da 10 tane ( ) ikilisi bulunur.
Sonuç olarak, tüm çözümler sayısı 23 + 10 = 33 olur.
2
n (n + 1)
biçimindeki sayıya
2
bir üçgensel sayı denir. Buna göre, a − b = 90 eşitliğini sağlayan kaç tane (a, b)
üçgensel sayı ikilisi vardır? (AÜMO 2009)
 ( + 1)
 ( + 1)
Çözüm :  =
ve  =
 (  ∈ N) eşitliklerinden,
2
2
¢
1¡ 2
90 =  −  =
 +  − 2 − 
2
1
90 =
( − ) ( +  + 1)
2
1 · 2 · 2 · 3 · 3 · 5 = ( − ) ( +  + 1)
Örnek 132 n bir pozitif tamsayı olmak üzere, a =
olur.  −  ve  +  + 1 sayılarının biri çift olduğunda diğeri tek olacağından, küçük
çarpan olan  −  sayısı 1  3 5 9 15 ve 4 değerlerini alabilir.  −  = 1 olursa,
++1 = 180; − = 4 olursa, ++1 = 45 v.b. Buradan  ve  dolayısıyla,
( ) ikilileri bulunur. Yanıt 6 olur.
Örnek 133 a, b ve c ∈ R ve m ∈ Z+ olmak üzer,
1 3
n −an2 −bn − c
m
ifadesi, her n ∈ Z için tamsayı olduğuna göre, m sayısının alabileceği kaç değer
vardır?
1
Çözüm : Her  ∈ Z için, 3 − 2 −  −  ifadesi tamsayı ise,

1
( + 1)3 −  ( + 1)2 −  ( + 1) − 

ifadesi de tamsayıdır. Bu iki ifadenin farkından,
µ
¶
3 2
3
1
 +
− 2  +
−−



elde edilir.
3
1
− 2 =  ve
−  −  =  denilirse, elde edilen


3 2
 +  + 

(∗)
71
Temel Bilgiler
ifadesinin de, her  ∈ Z için, tamsayı olması gerekir. Benzer şekilde,
3
( + 1)2 +  ( + 1) + 

ifadesi de tamsayı olmalıdır. (∗∗) ve (∗) ifadelerinin farkından,
6
3
++


elde edilir. Bu ifade de her  ∈ Z için tamsayı olmalıdır. Benzer düşünceyle,
µ
¶ µ
¶
6
3
6
3
6
( + 1) +  +
−
++
=





(∗∗)
ifadesi de tamsayı olmalıdır. Buradan,  pozitif tamsayısı 1 2 3 veya 6 olabilir.
2
Örnek 134 x2 = y 2 +2 denklemini sağlayan kaç (x, y, z) asal sayı üçlüsü
vardır?
2
Çözüm : Denklemi 2 −  2 = 2 biçiminde yazalım. Buradan,
( − ) ( + ) = 2
2
olacaktır. ( − ) ve ( + ) sayılarının biri tek ise diğeri de tek, biri çift ise diğeri
de çifttir. Sağ taraf 2’nin bir kuvveti olduğundan her ikisi de 2’nin bir kuvvetine eşit
olması gerekir. O halde,
 −  = 2 ve  +  = 2
2
−
diyelim. Bu denklemleri taraf tarafa toplarsak,
2 = 2 + 2
2
−
taraf tarafa çıkarırsak da
2 = 2
2
−
− 2
elde edilir.  ≥ 2 olması durumunda,  ve  sayıları çift sayı olacaklardır.   
olduğundan  ve ’den biri kesinlikle asal olmayacaktır. O halde,   2 olmalıdır.
 = 1 ise 2 = 21 + 2
2
−1
ve 2 = 2
 = 1 + 2
2
2
−1
−2
− 21 eşitliklerinden,
ve  = 2
2
−2
−1
bulunur.  bir tek asal ise,  sayısı 3’e bölünür. Bu durumda, ’in asal olması sadece
 = 3 iken mümkündür ki,  −  = 21 eşitliğinden,  = 1 olur. Yani  asal olmaz.
O halde,  bir çift asal, yani  = 2 olmalıdır. Böylece,  = 5 ve  = 3 elde edilir.
Sonuç olarak verilen denklem sisteminin tek (  ) asal sayı çözümü (5 3 2) olarak
bulunur.
Download